Morepedsfinal 2

You might also like

Download as pdf or txt
Download as pdf or txt
You are on page 1of 39

1. Which concept reinforces the development of a 11. When assessing a family, the nurse 22.

urse 22. The nurse is caring for a patient who has 32. Select the heart sound produced by vibrations
sense of trust for an infant? They can predict determines that the parents expect rules to be chosen to breastfeed her infant. Which statement within the heart chambers or in the major arteries
what is coming and their needs are consistently followed rigidly and unquestioningly. This style should the nurse include when teaching the from the back and forth flow of blood. Murmur
met of parenting is called? Authoritarian mother about breastfeeding problems that may
occur? If mastitis occurs, continue 33.Select from the following list the two basic
2. What childhood vaccine provides some 12. Parents ask the nurse if there was something breastfeeding while taking prescribed concepts used by Pediatric Nurses to foster family
protection against bacterial meningitis, epiglottitis, that should have been done during the pregnancy antibiotics and apply warm compresses centered care with children and their families:
and bacterial pneuomonia? Hib vaccine to prevent the child’s cleft lip. Which statement Enabling and Empowerment
should the nurse give as a response? The 23. A child with acetaminophen (Tylenol)
3. The nurse is assessing breath sounds on a child. malformation occurs at approximately 6 weeks poisoning has been admitted to the emergency 34. A child begins to blame his father for his
Which is a physiologically expected auscultated of gestation, there is no known way to prevent department. What antidote does the nurse parent’s divorce and has displayed intense anger
breath sound? Vesicular this anticipate being prescribed? N-acetylcysteine towards his father. In which age group of children
(Mucomyst) is this most likely to occur? School age
4. Children do not like the discomfort associated 13. A 3 month old dies shortly after arrival to the
with venous, arterial, and capillary punctures. emergency department. The infant has subdural 24. Exposure to which of the following 35. The parents of an infant with a cleft palate ask
Which of the following is the most appropriate and retinal hemorrhages but no external sign of communicable diseases pose a serious threat to the nurse “What follow-up care will our infant
pain reduction technique to use with most children trauma. The nurse should suspect: Shaken baby fetuses during the first trimester of pregnancy? need after the repair?” Which is an accurate
before these painful procedures? Apply EMLA syndrome Rubella response by the nurse? Your infant will need
cream topically over the site if time permits follow-up care with audiologists and
14. Select the vaccines usually given to children 25. At which age should the nurse expect an orthodontists
5. A parent brings a 6 month old to the pediatric at 2 months of age according to the immunization infant to begin smiling in response to pleasurable
clinic for her well baby examination. Her birth schedule. DTaP, IPV, PCV, HIB, RV stimuli? 2 months 36. A 6 y.o male is diagnosed with a heart
weight was 8.2 lbs. The nurse weighing the infant murmur that can be heard without the
today would expect her weight to be at least? 16 lb 15. Which of the following statements is true of X 26. One of the most common intestinal parasitic stethoscope. The nursing student is aware that this
linked recessive inheritance? Affected males pathogens in the United States acquired from a occurs at which of the following grade? VI
6. Along with birth, which of the following is a have symptoms of the disorder contaminated water source such as a lake or
significant risk predictor for infant mortality risk? swimming pool is? Giardia intestinalis 37. At what age should the nurse expect most
Gestational age 16. A newborn’s assessment reveal a fused digit infants to be saying Mama or Dada? 10 months
on the left hand. Which of the following is used to 27. What do morbidity statistics describe? The
7. A child has been diagnosed with giardiasis. define fused digits of the hands or feet? prevalence of specific illness in the population 38. The mother of a 3 month old breastfed infant
Which prescribed medication should the nurse Syndactaly at a particular time asks about giving her baby water because it is
expect to administer? Tinidazole (Tindamax) summer and very warm. What should the nurse
17. By which age should the nurse expect that an 28. A child is admitted to the hospital with an tell her? Fluids in addition to breastmilk are
8. A nurse is assessing a 2 ½ y.o toddler at a well infant will be able to pull to a standing position? undiagnosed rash. The best initial nursing action not needed
child visit. Which clinical finding should be 11 to 12 months of the pediatric nurse should be? Place child in
reported to the healthcare provider? Head contact precaution isolation 39. An infant with an isolated cleft lip is admitted
circumference exceeds chest circumference 18. Which of the following nonpharmacologic to the ICU for pre-op care. Which information
(hydrocephaly) interventions appears to be effective in decreasing 29. Which statement best describes the use of the should the nurse plan to discuss while educating
neonatal procedural pain? Oral sucrose and word ‘family’? A group that is what an the parents?(Select all) Use check support while
9. Select the rational for the relationship between nonnutritive sucking individual considers it to be feeding with special nipples, the infant may be
children having anemia and lead poisoning. restrained after surgery, and multiple
Children with anemia absorb lead more easily 19. Which term describes irregular areas of deep 30. The nurse is ready to perform a physical exam specialists will be assigned to infant’s care
blue pigmentation seen predominantly in infants on a 9 month old infant. Where should the nurse
10. A 7 month old patient with up-to-date of African, Asian, Native American, and Hispanic place the infant? On the parent’s lap 40. Which of the following symptoms are usually
immunization has a runny nose, low-grade fever, descent? Mongolian spots seen in a child with Rubeola (Measles)? Rash
and a paroxysmal cough that has worsened over the 31. The parents of a child diagnosed with an appearing on the 3rd day, Kolpik spots, and
past few days. The patient had coughed so hard 20. The Apgar score of an infant 5 minutes after autosomal recessive condition consult the nurse Conjunctivitis
that scleral hemorrhages are present. The most birth is 5. Which is the nurse’s best interpretation stating they want to have more children but are
likely diagnosis is? Pertussis of this? Resuscitation is likely to be needed worried about subsequent children also inheriting 41. What identified characteristics occur more
the disease. What information should the nurse frequently in parents who abuse their children?
21. According to Piaget, adolescents tend to be in provide to the parents? Each child has a 1 in 4 (Select all) Socially isolated, Few supportive
what stage of cognitive development? Formal risk of inheriting the disease relationships, and Single parent families
operational thought
42. The nurse is reviewing the Healthy People 202 55. A Pediatric Nurse Practitioner (PNP) in the 66. The parent of an 8.2 kg 9 month old infant is 77. The components of the FLACC scale include
leading health indicators for a child health peds clinic is assessing the reflexes of a 6 month borrowing a federally approved car seat from the cry, leg movement, facial expression and activity.
promotion program, Which are included in the old infant. Which of the following reflexes should clinic. The nurse should explain that the safest Consolability
leading health indicators?(Select all) Improve usually not be seen at this age? Startle way to put in the car seat is what? Rear facing in
imnization rates, Increase access to healthcare, back seat 78. The Pediatric nurse should begin screening for
Decrease tobacco use 56. Select the vaccines usually given to the lead poisoning when a child reaches which age?
children at 12 months of age according to the 67. Parents ask the nurse about the characteristics 12 months
43. Select the disorders from the list classified as immunization schedule: DTaP, HEP A, MMR of autosomal dominant inheritance. Which
inborn error of metabolism diseases.(Select all) statement is characteristic of autosomal dominant 79. The nurse is finishing her shift on the pediatric
Phenylketonuria, Galactocemia 57. The pediatric nurse is planning to educate and inheritance? Each child of a heterozygous unit. Because her shift is ending, which
empower a family to use the information provided affected parent has a 50% chance of being intervention takes top priority? Documenting the
44. Which is a sex chromosome abnormality that is to build their competence. This preventative affected care provided during her shift
caused by the absence of an X chromosomes in a measure is referred to as? Anticipatory guidance
female? 68. Choose the prevention strategies that have 80. Which of the following chromosomal
Turner 58. A 17 month old child should be expected to be significantly decreased fatalities for children. disorders seen in affected newborn causes
in which stage according to Piaget? (select all) Bicycle helmets, car restraints, and significantly shortened life span? Patau
45. Identify the age group of children per Erickson Preoperations smoke detectors Syndrome
in the development stage of industry vs inferiority:
6-12 59. Which of the following is the most consistent 69. Which of the following symptoms are usually 81. Statistically how many children in the U.S
and commonly used data for assessment of pain in seen in a child with chickenpox? (Select all) Rash experience mental health problems that affects
46. Which condition/behavior manifested by an 11 infants? Behavioral in 3 stages, Vesicles that break easily, slight their daily functioning? 1 out of 5 children
month old infant warrants further evaluation? fever
Unable to pull to a standing position 60. Which of the following is an important 82. Which of the following body systems can be
consideration when using the APPT pain rating 70. A foster parent is talking to the nurse about severely affected with an increased lead level in a
47. The nurse notes that a 12 month old infant who scale with children? Children color the area the health care needs for the child who has developing child? Hematologic and neurologic
weighed 6.5 lbs at birth now weighs 20.5 lbs. What with the color they choose to best describe their Foster children tend to have a higher than
is the nurse’s evaluation of the infant’s current pain normal incidence of acute and 83. A breastfed infant has just been diagnosed
weight? Infant’s weight is appropriate for the with galactosemia. The therapeutic management
age 61. In terms of gross motor development what 71. Which of the following reflexes usually of this include which? Stop breastfeeding the
should the nurse expect an infant age 5 months to disappear in a newborn around 3-4 months of infant
48. A parent reports to the nurse that her child has do? Turn from the abdomen to the back age? (Select all) Moro, Startle, Rooting
inflamed conjunctivae of both eyes with itching 84. A blood pressure cuff size that is too small for
and watery to thick stringy discharge. These 62. The parent’s of a 12 month old child ask the 72. What developmental achievements are a child will give which of the following reading?
manifestations suggest? Allergic conjunctivitis nurse if the child can eat hot dogs as do their other demonstrated by a 4 y.o child? (Select all) False high reading
children. The nurse’s reply should by based on Commonly has an imaginary playmate, Tends
49. Select an initial assessment included in the what? Hot dogs must be cut into small irregular to be selfish and impatient, Fears are common 85. What is the single most important factor to
Apgar scoring system? Color pieces to prevent aspiration consider when communicating with children?
73. Identify the age group of children per Child’s developmental level
50. A 2 y.o is scheduled to have a tonsillectomy. 63. Infant mortality is defined as death in which Erickson in the developmental stage of Autonomy
How would you educate the patient? Use picture of the following parameter? First year of life vs. Shame & Doubt. 1-3 86. The most important criterion on which to base
books and puppets and repeat explanations the decision to report suspected child abuse is
64. The Pediatric Nursing student is educating a 74. The Phenylketonuria blood screen test will be which of the following? Incompatibility between
51. You are observing a 5 month old for mother who plans to discontinue breast-feeding most reliable on a newborn if which of the the history and injury observed
developmental skills. Which of the following skills when the infant is 9 month old. The nursing following occurs initially? Received breast milk
would concern you if the infant was not student should advise her to include which foods or formula at least 24 hours before the test 87. The nurse suspects that a child has ingested
demonstrating? Turn head to locate sound in her infant’s diet? Iron rich formula only some type of poison. What clinical manifestation
75. Pertussis vaccination should begin at which of would be most suggestive that the poison was a
52. Which of the following is true regarding deaths 65. Select the viral communicable disease from the following ages? 2 months corrosive product? edema of the lips, tongue,
caused by unintentional injuries in childhood? the following list, which has the classic symptoms and pharynx
More deaths occur in males of photophobia, kolpik, spots on the buccal 76. At what age is it safe to give infants whole
mucosa, and a rash that begins on the face. milk instead of commercial infant formula? 12 88. Children may believe that they are responsible
53. A pregnant teen agrees to chromosome analysis Rubeola months for their parent’s divorce and interpret the
testing of her fetus. This is usually accomplished separation as punishment. At which age is this
through testing of which of the following? 89. Which statement below best describes family most likely to occur? 4 years
Amniotic fluid systems theory? Change in one family member
creates a change in other members
90. The nurse is caring for a child receiving a 100. What should a nursing intervention to 111. What is the leading cause of death in 122. A Pediatric nursing student, while assisting
continuous intravenous (IV) low dose infusion of promote parent-infant attachment include? children older than 1 year in the U.S.? in teaching nutrition to new parents, informs them
morphine for severe postoperative pain. The nurse Explaining normalcy of specific infant Complications from childhood unintentional that eating preferences are influenced primarily by
observes a slower respiratory rate and the child behaviors to parents injuries the family. At what age is lifelong eating habits
cannot be aroused. The most appropriate usually established? Age 3
management of this child is for the nurse to do 101. Identify the age group of children per 112. A lesion which begins on the trunk and
which first? Administer naloxone (Narcan) Erickson in the developmental stage of Initiative spread to the face and proximal extremities 123. Which of the following is the best method
vs Guilt. 3-6 describes which of the following communicable for performing a physical examination on a
91. Evidence based practice in clinical management diseases? Varicella toddler? From least to most intrusive
is grounded in which of the following? Cohort 102. Select the vaccines usually given to children
studies at 6 months of age according to the immunization 113. Which of the following characteristics 124. What do mortality statistics describe? The
schedule. DTaP, RV, HIB, PCV describes the child at high risk for childhood number of individuals who have died over a
92. Which is a sex chromosome abnormality that is injuries? Male with a high activity level and a specific period
caused by the absence of an X chromosomes in a 103. In the recovery room, the best immediate stressful home life
female? Turner post-op position for an infant who had cleft lip 125. A child has been admitted to the hospital
repair is? Supine with the head turned to the 114. Which of the following concepts provides a with a blood lead level of 42 mcg/dl. What
93. Identify the age group of children per Erickson side rational approach to decision making that treatment should the nurse anticipate? Initiation
in the developmental stage of industry vs. facilitates best practice in nursing care? Evidence of chelation therapy
inferiority? 6-12 104. Which of the following is a classic finding in Based Practice
infants diagnosed with Abusive Head Trauma? 126. After the family, which has the greatest
94. Phenylketonuria is a genetic disease that results Retinal hemorrhaging 115. A child has been diagnosed with giardiasis. influence on providing continuity b/w
in the body’s inability to correctly Which prescribed medication should the nurse generations? School
Phenylalanine 105. The nurse is teaching student nurses about expect to administer? Metronidazole (Flagyl)
newborn screening. Which statement made by the 127. Identify the age group of children per
95. Select an initial assessment included in the student indicates understanding of the teaching? 116. What is a function of brown adipose tissue Erickson in the developmental stage of Identity vs.
Apgar scoring system. Color, heart rate, reflexes, The newborn screening is now mandatory in (BAT) in newborns? Generates heat for Role Confusion. 13-21
muscle tone, and respiration all 50 states distribution to other parts of body
128. Which is the leading cause of death in infants
96. The nurse is conducting a teaching session for 106. When discussing discipline with the mother 117. When assessing a family, the nurse younger than 1 in the U.S? Congenital anomalies
parents on nutrition. Which characteristics of of a 4 y.o child, the nurse should include which of determines that the parents exert little or no
families should the nurse consider that can cause the following true statement? Parental control control over their children. This style of parenting 129. Select the vaccines usually given to children
families to struggle in providing adequate should be consistent is called? Permissive at 12 months of age according to the
nutrition? (Select all) Lower income, migrant immunization schedule: IPV, VAR, MMR, HEP
status, homelessness 107. The nurse is talking to a group of parents of 118. A rash which begins on the trunk, does not A
school-age children at an after-school program spread to the face but has sandpaper like quality,
97. Select the developmental milestones usually about childhood health programs. Which of the describe which of the following communicable 130. The nurse is having difficulty
seen in children during the toddler stage (1-3 following statements should the nurse include in diseases? Scarlet fever communicating with a hospitalized 6 yr old.
years). (Select all) Two to three word sentences, the teaching? Childhood Obesity can be Which technique should be most helpful? Provide
appears to be bowlegged and potbellied, prevented by lifestyle changes 119. What is the most common form of child supplies for the child to draw a picture
maltreatment? Child neglect
98. At what age can most infants sit steadily 108. A newborn’s assessment reveal an extra digit 131. Which of the following is the causative agent
unsupported? 8 months on both hands. Which of the following terms is 120. Which of the following chromosomal of scarlet fever? Group A β hemolytic
used to define extra digits of the hands or feet? disorders seen in affected newborns causes streptococci (GABHS)
99. A nurse is providing anticipatory guidance to Polydactaly significantly shortened life span? Edwards
the parents of a toddler. Which of the following Syndrome 132. An adolescent patient wants to make a
should the nurse include in the teaching? 109. The nurse is administering the first hepatitis decision about treatment options, along with his
Expect negative behaviors associated with A vaccine to an 18 month old child. When should 121. During a well-baby visit a parent asks the parents. Which moral value is the nurse
negativism and ritualism, Develop food habits the child returns to the clinic for the second dose nurse when she should start giving solid foods. displaying when supporting the adolescent to
that will prevent dental caries, and Expression of hepatitis A vaccination? After 6 months The nurse should instruct her to introduce which make decisions? Autonomy
of bedtime fears is common solid food first? Rice cereal
1.The nurse is assessing a 12 year old child with 8.The nurse notes that the parents of a critically 16.Transposition of the Great Vessel(TGV) is 24.A 4 y.o male is rushed to the emergency dpt
a foreign object in the left eye. What action ill child spend a lot of time talking to the parents incompatible with life.Prostaglandin E is given to during an acute severe prolonged asthma attack
should the nurse take? Remove a freely of another child who is seriously ill. They talk with children with this medical emergency to keep and is unresponsive to usual treatment. The
movable object with the pointed corner of these parents more than the nurses. How should which of the following shunts open in an infant condition is referred to as which of the following?
gauze pad moistened with water the nurse interpret this situation? Parent to with TGV? Ductus Arterlosus Status Asthmaticus
parent support is valuable
2. A 10 year old is seen in the developmental 17.The physician suggests that surgery be 25.A school nurse is educating a room full of 12-
clinic with the following symptoms moderate 9. A nurse in the Tb clinic is preparing to performed for patent ductus arteriosus(PDA) to 12 y.o females on health and hygiene issues. She
cognitive impairment, mitral valve prolapse, administer Directly Observed Therapy(DOT) to a prevent which complication? Pulmonary vascular included the following symptoms: high fever,
autistic like behaviors such as social pt with active Tb. Which of the following congestion vomiting, diarrhea, oliguria, and hypotension to
anxiety.Which of the diagnosis below do these medications will the nurse expect to administer? best describe which illness? Toxic Shock
symptoms best describe? Fragile X Syndrome Isoniazid 18.A 4 y.o child seen in the ER has symptoms of Syndrome(TSS)
Kawasaki Disease(KD). Of the symptoms listed
3. A 5 year old has bilateral eye patches in place 10.What strategy should the nurse recommend to below which can potentially develop and pose a 26.The presence of which pair of factors is a good
after surgery yesterday morning. Today he can parents, to get the siblings of a child with special high risk for children with KD? Coronary artery predictor of a fluid deficit of at least 5% in an
be out of bed. Which of the following is the most needs involved in the child’s care? Request the aneurysm infant? Capillary refill of greater than 2 secs and
important nursing intervention? Orient him to sibling’s help to design strategies that will work weight loss
his immediate surroundings at home 19.Which of the following actions by the school
nurse is most important in the prevention of 27.A nurse is reading a physician’s progress notes
4. A 14 month old boy is hospitalized with 11.A parent of a child with a disability expresses rheumatic fever(RF)? Refer children with sore in the client’s record and reads that the physician
dehydration. He is inconsolable, screaming, and guilt to the nurse about the child’s condition. throats for throat cultures has documented ‘insensible fluid loss of
rejecting your physical contact. What best What nursing intervention is most appropriate in approximately 800 ml daily.’ The nurse
describes his response? Separation Anxiety- this situation? Explore the parent’s adjustment 20.A 5 y.o is admitted with bacterial pneumonia. understands that this type of fluid loss can mainly
PROTEST PHRASE responses What signs and symptoms should the nurse occur through? Skin
expect to assess with this disease process? Fever,
5.The Peds nurse receives a phone call from the 12. What behavior seen in children should be cough, chest pain 28.A 6 y.o male in the peds ER has a history of
Pediatrician for a 10 y.o pt who weighs 40 kg for addressed by the nurse who is providing care to a fluid loss and dehydration over the past 12 hrs.
Ceftazidime(Fortaz) 1.5 grams q 8 hrs IV. The child with chronic illness? An adolescent who is 21.Which of the following test aids in the The nursing assessment should include close
therapeutic dosage range is 90-150 mg/kg/24 becoming more dependent on family diagnosis of cystic fibrosis(CF)? Sweat test, stool monitoring for the possibility of which of the
hrs. What would be the best nursing action? for fat, chest x-ray films following? Impending shock
Administer the medication bc it is within the 13.The nurse understands that a preschool child
therapeutic dose range may react to death with what reaction? Joking 22.A child diagnosed with Coarctation of Aorta is 29.A 10 month old infant has acute otitis media
scheduled for a f/u visit. While assessing the (AOM) and is being treated with oral antibiotics
6.The nurse is preparing to administer a liquid 14.The practitioner has ordered a liquid oral pediatric nurse would expect to find which of the for the 3rd time in the past 8 months. What
medication by a nasogastric feeding tube. What antibiotic for a toddler with otitis media. The following symptoms? should the nurse include in the discharge
is the first thing the nurse should do? Check the prescription reads 1 ½ tsp 4x/day.What should Absent or diminished femoral pulses teaching to the infants parents?
pH of the gastric aspirate the nurse consider in teaching the mother how to The infant will need a f/u visit for possible
give the medicine? Disposable calibrated oral 23.Select the parts of the fetal circulation of the tympanostomy procedure and tubes
7.What drug is an anglotensin-converting syringe pediatric heart in the correct order as blood flows
enzyme(ACE) inhibitor? Captopril(Capoten) from the placenta to the fetus: 30.A parent prepares to administer pancreatic
15.Tetralogy of fallout consists of pulmonic Placenta-umbilical vein-ductus venosus-inferior enzymes to an infant with cystic fibrosis. As per
54. Parents of a toddler ask the nurse “Should we stenosis, overriding aorta, ventricular septal vena cava-R atrium education she received the best action to
set rules for our child as part of a discipline defect, and which of the following? Right administer the enzymes is which of the following?
plan?” Which is an accurate response by the ventricular hypertrophy 134. A parent brings a toddler, age 19 months, to Increase the dose of pancreatic enzymes if infant
nurse? Set clear and reasonable rules and the clinic for a regular check-up. What should the is having frequent bulky stool
expect the same behavior regardless of the 133. A couple has given birth to their first child nurse expect to find when palpating the toddler’s
circumstances with a recessive disorder. The genetic counselor fontanels? 110. An 8 month old infant is sitting contentedly
tells them that the risk of recurrence is one in four. Closed posterior and anterior fontanels on the mother’s lap. The nurse is preparing to
Which statement is a correct interpretation of this perform a well baby checkup. Which of the
information? The risk factor remains the same following steps should the nurse do first?
for each pregnancy Auscultate heart and lung sounds
31.A 2 y.o has been placed in contact isolation 38.A nurse in the Tb clinic is preparing to teach a 45.A 13 month old child seen in the peds clinic 55.An 18 month old child is seen in the clinic with
because of diagnosis of respiratory syncytial pt with active Tb about the medications. Which of shows symptoms of tachycardia, tachypnea, cap otitis media(OM). Oral amoxicillin is prescribed.
virus(RSV)bronchlolitis. The father questions why the following medications will the pt need to take refills of 3 sec, and slight weight loss since the last Instructions to the parent should include which of
the staff is wearing masks and gowns every time initially?(SELECT ALL)Rifampin, Isoniazid, visit. Which level of dehydration is the child the following?Medication may take 24-48 hrs to
someone comes into the room. What is the best Ethambutol entering? Moderate make symptoms subside
response by the nurse? It is important for the
staff to wear the equipment to prevent 39.Which of the following are part of the stages of 46.A 6 y.o in the peds Er has a diagnosis of 56.What clinical finding may be present in an
spreading it to others grief an individual may experience while dealing hypovolemic shock. What are the most frequent older child with Coarctation of the Aorta?
with the death of a child?(SELECT ALL) causes of this type of shock in children? Blood High blood pressure in the upper extremities
32.During the assessment of a 9 y.o child the Ambivalence, Denial, Anger Loss
nurse notes excessive drooling, the child is 57.A 5 y.o is seen in the urgent care clinic with the
fearful refuses to lay down. Which condition 40.The clinic nurse is assessing an infant. What 47.Which behavior indicates an 18 month old is following history and symptoms:sudden onset of
does the nurse suspect? Epiglottitis are the early signs of cognitive impairment the developing a non-adaptive reaction to a severe sore throat after going to bed, drooling
nurse should discuss with the health care prolonged hospitalization? Ignores mother when and difficulty swallowing, axillary temperature of
33.The nurse caring for a 6 y.o child who has provider?(SELECT ALL) she arrives 102.2 F(39.0 C), clear breath sounds, and absence
undergone cardiac catheterization 3 hrs ago Unresponsiveness to the environment at 12 of cough. The child appears anxious and is
notes the dressing and bed saturated with blood. months of age, unable to speak 2-3 words at 24 48.What finding is the most reliable guide to the flushed. Based on these symptoms and history,
Which initial action should the nurse take? Apply months of age, head lag at 11 months of age adequacy of fluid replacement for a small child the nurse anticipates a diagnosis of?
pressure above the insertion site with burns? Urinary output of 1-2 ml/kg of body Acute epiglottitis
41.Select the surgical procedures usually done in weight/hr
34.Which of the following medication is children to correct heart defects from the 58.Select from list below a reason for ‘Innocent’
commonly used to treat Kawasaki’s Disease(KD)? following list(SELECT ALL) 49.A 2 y.o child is receiving digoxin(Lanoxin). The heart murmurs in infants: Fever
IVIG Modified Blalock-Taussig Shunt nurse should notify the practitioner and withhold
Bidirectional Glenn Shunt the medication if the apical pulse is less than 59.A pediatric nurse who is caring for a 12 y.o pt
35.A 10 y.o sustained a full thickness 3rd degree which rate? 90 beats/min with septic shock perceives a potential
burn on his left arm during a BBQ. What will be 42.A 15 y.o male referred to a cardiologist is complication of fluid resuscitation upon which of
required for healing from this injury? diagnosed with Infective Endocarditis. Which of 50.A 3 month old with a hypercyanotic spell. the following? Rales and rhonchi by auscultation
Autografting the following symptoms are usually seen with this What should be the nurse’s first action?
condition?(SELECT ALL) Place the child in knee chest position 60.Of the statements below what statement best
36.What intervention should be included in the Murmur, Osler nodes, Janeway spots represents infectious mononucleosis?
nursing care of a child with autism spectrum 51.What structural defects constitute tetralogy of Lymphadenopathy is a common characteristic
disorder(ASD)? Decrease auditory and visual 43.The healthcare provider prescribes Fallot? Aortic stenosis, ventricular septal defect,
simulation vancomycin 200 mg per intravenous overriding aorta, right ventricular hypertrophy 61.A child who has a cardia catheterization is
piggyback(IVPB) q 6 h for a child with CF. The being discharged. What should the nurse include
37.Patients, parents, or legal surrogates must pharmacy send the meds to the unit in a 150 ml 52.What sign/symptom is a major clinical in the discharge instructions to the parents? Give
receive sufficient information on which to make bag with directions to run the medication over manifestation of rheumatic fever(RF)? sponge baths for 3 days
informed health care decision. Informed consent 120 mins. What ml/hr will the nurse set the IV Polyarthritis to prevent infection
should include which of the following?(SELECT pump to run the medication over 120 mins? 75
ALL) ml/hr 53.What child has a cyanotic congenital heart 62.A nurse is reviewing the laboratory values of a
Nature of the illness or condition defect?2 month old with tetralogy of Fallot child with rheumatic heart disease. Which finding
Proposed care or treatment 44.A 46 lb 4 y.o pt is ordered to have does the nurse conclude is related to this
Potential risks, benefits, or alternatives acetaminophen 10 mg/kg/dose oral suspension. 54.A child is recovering from Kawasaki’s condition?
Medication bottle is labeled 300 mg/2 ml. Disease(KD). The child should be monitored for Positive antistreptolysin titer
Calculate the dose to be administered. 3.1 which? Electrocardiograph(ECG) changes
63.The Pediatric Nurse assigned to an 8 y.o is 72.After returning from cardiac catheterization 81.The nurse is assessing coping behaviors of a 91.An initial action of the nurse in caring for a
aware that the child’s fluid need will be increased the nurse determines that the pulse distal to the family with a child with a chronic illness. What child with Status Asthmaticus is which of the
in which of the following circumstances? Shock catheter site is weaker. How does the nurse indicates avoidance coping behaviors? (SELECT following? Administer beta 2 agonists as ordered
respond? Observe this as a normal finding and ALL) Refuses to agree to treatment, Avoids staff,
64.What type of dehydration occurs when the record the data family members, or child, Is unable to discuss 92.A 2 y.o child diagnosed with tetralogy of Fallot
electrolyte deficit exceeds the water deficit? possible loss of the child becomes upset, crying, and thrashing around
Hypotonic dehydration 73.What drug is an angiotensin-converting when a blood specimen is obtained. The childs
enzyme(ACE) inhibitor? Enalapril(Vasotec) 82.The nurse is interpreting a tuberculin skin test. color becomes blue and respiratory rate increases
65.A 10 y.o sustained a superficial 2nd degree If the nurse finds a result of an induration 5 mm to 44 bpm. Which of the following actions would
partial thickness burn on his left arm during a 74.Which of the following is appropriate for a or larger in which should the nurse document this the nurse do first?Place the child in knee to chest
BBQ. What is the expected time of healing for this hospitalized infant?Rattles, squeaking toys, finding as positive?(SELECT ALL) A child receiving position
injury? 3-4 weeks mobile toys immunosuppressive therapy, A child with a HIV
infection, A child living in close contact with a 93.Select from the list below a reason for
66.The nurse understands that a preschool age 75.A school age child with CF takes 4 enzyme known contagious case of tuberculosis ‘innocent’ heart murmur in infants: Anemia
child may react to death with what reaction? capsules with meals. The child is having 4 or 5
Showing regressive behaviors bowel movements/day. The nurses action in 83.What does the nurse recognize as physical 94.A nurse is evaluating the management of a
regard to the pancreatic enzymes is based on the signs of approaching death?(SELECT ALL) Mottling child with a history of asthma. Which statement
67.A parent requests info on her infants hearing. knowledge that the dosage is what? Needs to be of skin, Cheyne-Stokes respirations, decreased from the mother quires further investigation?
Which of the following should the nurse say is a increased to decrease the number of bowel appetite and thirst When my child has an attack she usually has to
clinical manifestation of hearing impairment in an movements/day use her rescue inhaler 4x before her breathing
infant less than 12 months old? Responds to loud 84.Which statement is true concerning Fragile X improves
noises as opposed to voice 76.A school age child has asthma. The nurse syndrome? The disease has no cure and
should teach the child that if a peak expiratory pharmaceuticals are used to manage symptoms 95.Which of the following would be involved in
68.The Cochlear implantation is a safe surgical flow rate is in the yellow zone that means that the the nursing care of a child with an atrial septal
technique used to treat which of the following asthma control is what? 85.Which of the following describes moderate defect? Preparing the child and family for
types of hearing loss in children? 50-79% of a personal best and needs and persistent asthma symptoms? Symptoms seen on cardiac catheterization
Sensorineural increase in the usual therapy a daily basis
96.The Pediatric Nurse assigned to an 8 y.o is
69.What intervention should be included in the 77.What drug is usually given first in the 86.What is a principle of palliative care that can aware that the childs fluid need will be decreased
nursing care of a child with autism spectrum emergency treatment of an acute severe asthma be included in the care of children? Uses a in which of the following circumstances?
disorder(ASD)? Provide individualized care episode in a young child? Short acting beta2- multidisciplinary approach Increased Intercranial Pressure
agonists
70.The nurse is performing a physical assessment 87.When teaching a mother how to administer 99.An 8 month old child seen in the peds clinic
on a 7 y.o child. The parents state that the child 78.What description applies to fragile X eye drops where should the nurse tell her to with Otitis Media w Effusion(OME) what is the
has trouble seeing the board at school. What syndrome? Second most common genetic cause place them? Place in the small sac that is formed best treatment option for this child?
visual impairment should the nurse suspect? of cognitive impairment when the lower eyelid is pulled down Tympanostomy tubes
Myopia or nearsightedness
79.A 10 y.o in the Peds ER has symptoms of 88.What is the best method to verify the 100.A 2 month old seen in peds clinic has
71.When caring for the intravenous(IV) site on a severe dehydration. What are signs of impending placement of a nasogastric tube before each use? symptoms of tachypnea, retractions, anorexia,
child, the nurse should take which action? Flush shock which needs to be monitored for in this Aspiration of stomach contents apneic spells, copious nasal secretions and
the site with normal saline per protocol child?(SELECT ALL) wheezing. Which of the following do these
Tachycardia, cool skin, hypoxemia 89.Verbal informed consent via the telephone for symptoms best describe?
97.The nurse is teaching a group of female a minor child is appropriate when the parent is RSV Bronchlolitis
adolescents about toxic shock syndrome and the 80.Which of the following are clinical not available in person. Which of the following is
use of tampons. What statement by a participant manifestations of a hearing impairment usually true in this case? 101. Of the statements below what statement
indicates a need for additional teaching: I can use seen in infants?(SELECT ALL) Absence of babbling The verbal consent is required to be in the best represents infectious mononucleosis?
a superabsorbent tampon for more then 6 hrs by 7 months, lack of startle reflex, responds to presence of 2 witnesses Clinical signs and symptoms and blood tests are
loud noises as opposed to the voice both needed to establish the diagnosis
98.An 11 month old child seen in the Peds clinic is 90.A child with heart failure is in Lanoxin(digoxin).
diagnosed with Isotonic Dehydration. Which of The laboratory value a nurse must closely monitor
the following si true of Isotonic Dehydration? is which? Serum Potassium
Electrolyte and water deficits are in balanced
proportions
102.Which of the following is usually affected in 5.What factor is most important for parents
cystic fibrosis, resulting in excess multisystem implementing do not resuscitate(DNR) orders?
mucus build up that is difficult to clear? Exocrine Acknowledgement by health care team that
Gland child has no realistic chance for cure

103.What are common respiratory symptoms 6.Autism is a complex developmental disorder.


dying children experience? (SELECT ALL) The diagnostic criteria for autism include delayed
Cough, wheezing, shortness of breath or abnormal functioning in which area with onset
before age 3 years?
104.Provider order: Augmentin 375 mg po q8h. Ability to maintain eye contact
Supply 75 ml bottle of reconstituted Augmentin
250mg/5ml. How many ml per dose? 7.5 ml 7.The nurse wants to provide a 3 month old infant
with stimulation. What is the best toy for the
105. The nurse is giving discharge instructions to nurse to provide?
the parents of a 5-year-old child who had a Mobile with a music box
tonsillectomy 4 hours ago. Which of the following
statements by the parent would indicate a 8.Select from the following list the maximum
correct understanding of the teaching? "I can use volume of solution that should be administered in
an ice collar on my child for pain control along a single IM site to an infant or a small child.
with analgesics." 1 ml

106.What type of shock is characterized by a 9.The nurse is assessing the coping behaviors of
hypersensitivity reaction causing massive the parents of a child recently diagnosed with a
vasodilation and capillary leaks, which may occur chronic illness. Which of the following should the
with drug or latex allergy? Anaphylactic Shock nurse consider an ‘approach behavior’ that results
in parental movement toward
107. A health care provider prescribes Osmitrol adjustment?Anticipating future problems and
(mannitol), 0.5 g/kg intravenously (IV) now for a seeking guidance and answers
child with minimal change nephrotic syndrome.
The child weighs 10 kg. The medication label 10.A 1 month old is admitted to the hospital. The
states: “Osmitrol (mannitol) 20 g/100ml”. The infant’s mother is 17 y.o and single and lives with
nurse prepares to administer the dose. How her parents. Who signs the consent form for the 1
many ml will the nurse prepare to administer the month old infant? The infant’s mother
dose? 25 ml
11.During a respiratory assessment, the nurse
1.A child’s chart indicates Astigmatism. For what notes a sinking in of soft tissue relative to the
should the nurse assess? Unequal curvatures in cartilaginous and bony thorax. What is the term
refractive apparatus for this finding?
Retractions
2.A school nurse decides to initiate a safety
program for increasing the use of bicycle helmets. 12.What is the best explanation for using pulse
The program is an example of: Primary oximetry on young children to determine oxygen
prevention saturation?
Pulse oximetry is noninvasive
3.The nurse should expect the apical heart rate of
a stabilized neonate to be which of the following 13.Secondary prevention for cognitive
ranges? impairment includes what activity? Newborn
120-160 beats/min screening for treatable inborn errors of
metabolism
4.Select the statement that is true concerning
upper respiratory infections(URI) in children 14.When giving oral liquids from a syringe to an
under 2 years old: Over the counter medications infant or child, which is the best procedure to
are not recommended for treatment of the use? Place the tip of the syringe into the pocket
common cold formed between the lower teeth and cheek
15.A 2 y.o is scheduled to have a tonsillectomy. 26.Families progress through various stages of
How would you educate the pt?Use picture reactions when a child is diagnosed with a
books and puppets and repeat explanation chronic illness or disability. After the shock
phase, a period of adjustment usually follows.
16.What condition is defined as reduced visual This is often characterized by what response?
acuity in one eye despite appropriate optical Guilt and anger
correction? Amblyopia
27.What behavior seen in children should be
17.When planning care for an 8 y.o boy with addressed by the nurse who is providing care to
Down syndrome, the nurse should: a child with chronic illness? A preschooler who
Assess the child’s current developmental level refuses to participate in self-care
and plan care accordingly
28.A 7 y.o child is in the end stages of cancer.
18.At which age do most children have an adult The parents ask you how they will know when
concept of death as being inevitable, universal, death is imminent. What physical sign is
and irreversible? 9-11 y.o. indicative of approaching death? Difficulty
swallowing
19.The parents of a 3 y.o admitted for recurrent
diarrhea are upset that the practitioner has not 29.The nurse is preparing to give acetaminophen
told them what is going on with their child. What (Tylenol) to a child who has a fever. What
is the priority intervention for this family? nursing action is appropriate? Check to be sure
Help the family develop a written list of specific the Tylenol dose does not exceed 15mg/kg
questions to ask the practitioner
30.The potential effects of chronic illness or
20.The nurse should expect a toddler to cope disability on a child’s development vary at
with the stress of a short period of separation different ages. Which of the following is a threat
from parents by displaying what? Regression to a toddlers normal development? Hindered
mobility
21.Which of the following age range of children
qualify to be included in Early Intervention 31.A first-time mother asks you what type of toy
programs? Birth-3 would be best for her 2 ½ y.o. Your response is?
Push-pull toys, large balls, and stuffed animals
22.What action should the school nurse take for
a child who has a hematoma (black eye) with no 32.Which of the following should be included in
hemorrhage into the anterior chamber? the nursing care of a child with autism spectrum
Apply ice for the first 24 hrs disorder(ASD)? Provide a structured routine for
the child to follow
23.A 7 y.o child tested positive for Streptoccal
Pharyngitis, but she’s allergic to Penicillin. Which 33.A pediatric nurse is caring for a child who is
of the following antibiotics is indicated as a terminally ill and expected to die. Which of the
treatment option? Erythromycin following procedures is usually no longer
necessary during the terminal phase? Measure
24.The nurse assesses the respiratory status of vital signs
an infant. Which finding should be of most
concern to the nurse? Expiratory grunting

25.A nurse in the Tb clinic is preparing to


administer Directly Observed Therapy(DOT) to a
pt with active Tb. Which of the following
medications will the nurse expect to administer?
Rifampin
1. Which of the following is standard therapy used 11. A 3 y.o with a Wilms tumor is returning to the 21. The nurse is preparing to administer a unit of 30. A parent asks the nurse what would be the first
in the care of children with Osteogenesis unit after surgery to remove the tumor. Which of packed red blood cells to a hospitalized child. indication that acute glomerulonephritis was
Imperfecta? Bisphosphonate Therapy and the following is the highest post-op priority for the What are appropriate actions that applies to improving. What would be the nurse’s best
physical therapy nurse? Monitor vital signs especially blood administering blood? Select All. response? Urine output will increase
pressure (b/c tumor of kidney) -Administer the first 50 ml of blood slowly and
2. An infant was assessed in peds clinic with the stay with the child 31. A child seen in renal clinic is diagnosed with
following symptoms: visible peristalsis, failure to 12. A common side effect of corticosteroid -Administer blood through an appropriate filter Vesicoureteral Reflux (VUR). Primary reflux
thrive, an infant who is ‘always hungry’, (prednisone) therapy for a 11 y.o with a cancer -Infuse blood within 4 hours usually occur as a result of which of the following?
dehydration. What is the likely diagnosis? Pyloric diagnosis is which of the following? Large Congenital anomaly
Stenosis abdomen with striae 22. The nurse is caring for an infant with suspected
pyloric stenosis. Which clinical manifestations 32. A 12 y.o child is seen in specialty clinic has ill
3. A 4 y.o male seen in the peds specialty renal 13. Which of the following describes the would indicate pyloric stenosis? Select All. fitting clothes, a rib hump and hip asymmetry,
clinic has symptoms of end stage renal disease and pathophysiology of leukemia? Unrestricted -Non-bilious vomiting and weight loss recently noticed by parents. Which of the
visual disturbances. Which of the following is likely proliferation of immature white blood cells -Projectile vomiting following is the likely diagnosis of this child?
his diagnosis? Alport Syndrome (WBCs) -Olive-shaped mass above umbilicus Scoliosis

4. The parent of a child with growth hormone (GH) 14. A 5 y.o male diagnosed with moderate 23. The nurse is assessing a 4 y.o boy in the 33. Which of the following musculoskeletal
deficiency is learning to administer GH therapy. dehydration should be given Oral Rehydration pediatric clinic. Which behaviors by the client disorders is associated with morning stiffness that
When is the best time for the parent to administer Solution by the nurse or parent with which of the would concern the nurse for possible Duchenne improves with activity? Juvenile Idiopathic
the GH to the child? At bedtime using subQ following rehydration therapy? 100ml/kg/4 hrs muscular dystrophy? Select all. arthritis
needle -Frequent trips and falls at home
15. A 10 y.o patient is undergoing hemodialysis -Places hands on thighs to push up to stand 34. A school age child with celiac disease asks for
5. A 10 y.o female seen in specialty peds clinic has with an internal AV fistula in place. What do you -Walks on tiptoes and has disproportionately guidance about snacks that will not exacerbate the
a diagnosis of Spina Bifida. On examination the do to prevent anxiety and pain associated with the large calves disease. What snack should the nurse suggest?
peds nurse observes dark tufts of hair at the lumbar shunt? Apply EMLA cream over fistula Popcorn
sacral region. Which of the following is the child’s 24. A newborn seen in endocrinology specialty
diagnosis? Spina Bifida Oculta 16. An 18 month old male is scheduled for surgery clinic is diagnosed with hypothyroidism. Which of 35. A 14 y.o is admitted to the emergency room
to reposition his undescended testes. What is this the following are common symptoms of with a fracture of the right humerus epiphyseal
6. The nurse reviews the record of a newborn infant child’s diagnosis? Cryptorchidism hypothyroidism as seen in newborns? Select all. plate through the joint surface. What information
and notes that a diagnosis of esophageal atresia with Weight gain, dry skin, hair loss does the nurse know regarding this type of
tracheoesophageal fistula is suspected. The nurse 17. A 2 y.o child is on prednisone for minimal fracture? This type of fracture requires different
expects to note which MOST LIKELY sign of this change nephrotic syndrome (MCNS). Which of the 25. The nurse is caring for an 18 month old with management to prevent bone growth
condition documented in the record? Choking with following indicates the effectiveness of prednisone rotavirus. Which of the following clinical complications
feedings therapy? Diuresis occurs as urinary protein manifestations would the nurse expect to observe?
excretion diminishes Mild fever and vomiting followed by onset of 36. After a pyloromyotomy, a week old infant is
7. While educating a new 7 y.o diabetic patient and watery stools prescribed feedings of 1 to 2 oz of Pedialyte q 3
the parents, the pediatric nurse includes which of 18. An 11 y.o child was just admitted to the hrs then advance to breastmilk ad tolerated
the following times during the day when the child’s hospital for observation after a head injury. What 26.Which of the childhood cancers listed below postoperatively. The nurse should advance the
blood sugar level needs to be monitored? Check clinical manifestation is most essential for the have a genetic link as a causative factor? feeding if which occurs? The infant is taking the
blood glucose levels ac and hs nurse to assess to detect early signs of a Retinoblastoma Pedialyte without vomiting or distention
neurological condition? Level of consciousness
8. A nurse evaluates discharge teaching as 27. A 12 y.o child is admitted to the PICU in acute 37. The nurse is evaluating the laboratory results
successful when the parents of an infant with a 19. A toddler with symptoms of sudden renal failure (ARF). The nurse prepares to of a stool sample. What is a normal finding? The
ventriculoperitoneal shunt insertion identify which inconsolable screaming or crying, drawing up of administer which of the following medication to laboratory reports a negative guaiac
sign as signaling a blocked shunt? Tense fontanel the knees to the chest, vomiting, and a tender rapidly provoke a flow of urine? Mannitol
and increased head circumference distended abdomen will probably be diagnosed (Osmitrol) or furosemide (Lasix) or both 38. What is an important consideration for the
with which of the following diseases? identification and treatment of cryptorchidism in a
9. A nurse is caring for an infant with Intussusception 28. What statement is an advantage of a peritoneal child by age 2 years? Treatment is necessary to
myelomeningocele scheduled for a surgical closure dialysis compared with hemodialysis? It is easy to maintain the ability to be fertile when older
in the morning. Which interventions should the 20. Of the following which are possible signs of learn and can be done at home
nurse plan for the care of the myelomeningocele Cerebral Palsy (CP) ? Select All. 39. A 2 y.o child has simple constipation and is
sac? Cover with a sterile, moist, nonadherent -Poor head control after age 3 months 29. A 3 y.o is admitted to the peds unit with brought to the clinic for evaluation. The
dressing -Persistent primitive reflexes minimal change nephrotic syndrome. What clinical therapeutic plan initially includes: Dietary
-Feeding difficulties manifestations are usually seen with this modification
10. What is a common clinical manifestation of diagnosis? Massive proteinuria,
juvenile hypothyroidism? Dry skin hypoalbuminemia, and edema
40. Diabetic ketoacidosis (DKA) results from an 50. A toddler is admitted to the pediatric unit with 59. The pediatric version of the Glasgow Coma 70. When does idiopathic scoliosis become most
excessive accumulation of which of the following? presumptive bacterial meningitis. The initial Scale is used on a 3 y.o male after falling from a noticeable? During the preadolescent growth
Ketone bodies from fat metabolism orders include isolation, intravenous access, tree in his back yard. A score of 3 on his chart is spurt.
cultures, and antimicrobial agents. The nurse associated with? Poor outcome or Comatose
41. A school age child with acute diarrhea and mild knows that the antibiotic therapy will begin: state 71. Parents of a newborn with ambiguous genitalia
dehydration is given oral rehydration solution When the medication is received from the want to know how long they will have to wait to
(ORS). The child’s mother calls the clinic nurse pharmacy 60. The following symptoms describes which of know whether they have a boy or girl. The nurse
because he is also occasionally vomiting. The nurse the conditions listed below: Multiple fractures at answers the parents based on what knowledge?
should recommend which of the following? 51. A 12 y.o is being treated with Buck’s Traction. birth, growth failure, autosomal recessive Gender assignments involves collaboration
Continue to give ORS frequently in small What are important nursing interventions for this inheritance? Osteogenesis Imperfecta between the parents and a multidisciplinary
amounts client? Verify that the skin traction maintains team./Gender assignment involves a complex
the extremity in an extended position 61. What is the most common cause for the decision-making process
42. Erythema Migrans is an annular rash commonly overproduction of growth hormones in
seen with which of the following infections? Lyme 52. A toddler is admitted to the hospital and report acromegaly? Pituitary adenoma 72. A 6-month-old infant with Hirschsprung disease
Disease leg pain an fever. Assessment reveals the toddler is scheduled for a temporary colostomy. What
is pale with body bruises. The health care provider 62. A newborn seen in peds renal clinic receives a should postoperative teaching to the parents include?
43. A child has been diagnosed with scabies. Which suspects acute lymphoblastic leukemia (ALL). diagnosis of Alport Syndrome. Which of the Assessing bowel function.
statement by the parent indicates understanding of The nurse will inform the parents that following should be included in the counseling
the nurse’s teaching about scabies? Everyone who confirmation of the disease will be determined by given to the parents? Encourage parents to seek 73. A 6-month-old infant with Hirschsprung disease
has been in close contact with my child will need which test? Bone marrow biopsy genetic counseling is scheduled for a temporary colostomy. What
to be treated should postoperative teaching to the parents include?
53. The student nurse is aware that the Ketogenic 63. Duchenne Muscular Dystrophy (DMD) has Skin and stoma care.
44. After a tonic clonic seizure, what symptoms diet is used as a therapy for seizure control in which of the following inheritance patterns?
should the nurse expect the child to experience? children. Which of the following best describes X linked recessive trait 74. Which of the following pharmacology therapy is
Lethargy and confusion the Ketogenic diet? High fat, low carb, and used to treat infants and children with
adequate protein diet 64. A 13 y.o male has a femoral shaft fracture. Gastroesophageal Reflux Disease (GERD)? Zantac
45. Which of the following parameters would the The nurse recognizes that the approximate healing (Ranitidine)
nurse monitor to evaluate the initial effectiveness of 54. Children with steroid induced Cushing time for this adolescent’s fracture is how long?
thickened feedings for an infant with syndrome should receive their medication at 8-12 weeks 75. The nurse is assisting a child with celiac disease
gastroesophageal reflux (GER)? Vomiting which of the following times to lessen the to select foods from a menu. What foods should the
Cushingoid features? Give the drug: Early in the 65. A nurse evaluates discharge teaching as nurse suggest? Corn on the cob with butter
46. Of the following diagnosis, which would the morning upon awakening successful when the parents of a school aged child
nurse identify as a priority for the infant with with a ventriculoperitoneal shunt insertion identify 76. The nurse is evaluating the laboratory results of
tracheoesophageal fistula (TEF)? Risk of injury 55. Staging criteria exist for most tumors. Which which sign as signaling a blocked shunt? a stool sample. What is a normal finding?
related to increased potential for aspiration of the following relates to the meaning staging? Irritability and increasing difficulty swallowing Stool pH 7.0
The higher the stage, the poorer the prognosis
47. A nurse is teaching clients in an outpatient 66. Which of the following P’s will the pediatric 77. A child with hypopituitarism is being started on
facility about the use of insulin to treat type 1 56. A 2 y.o is hospitalized with suspected nurse assess on a 10 y.o male after a recent growth hormone (GH) therapy. Nursing
diabetes mellitus. For which of the following types intussusception. Which finding is associated with fracture of the femur to determine the extent of his considerations should be based on which
of insulin should the nurse tell the clients to expect intussusception? Currant jelly stools injury? Select all. Pallor, pain, paresthesia knowledge? Replacement therapy requires daily
a peak effect 2 to 5 hrs after administration? subcutaneous injections
Regular insulin 57. The pediatric nurse is preparing to admit a 5 67. The nurse is planning care for a school-age
y.o with spina bifida cystica that was below the child with bacterial meningitis. What intervention 78. A 20-kg (44-lb) child in ketoacidosis is admitted
48. The nurse is doing a neurologic assessment on a second lumbar vertebra. What clinical should be included? Keep environmental stimuli to the pediatric intensive care unit. What order
2 month old infant after a car accident. Moro, tonic, manifestations of spina bifida cystica below the to a minimum. should the nurse not implement until clarified with
neck, and withdrawal reflexes are present. The second lumbar vertebra should the nurse expect to the physician? Begin intravenous line with D5
nurse should recognize that these reflexes suggest observe? Overflow incontinence with constant 68. As part of the diagnostic evaluation of a child 0.45% normal saline with 20 mEq of potassium
which of the following? Neurologic health dribbling of urine with cancer, biopsies are important for staging. chloride. (0.9% normal saline should be given).
What statement explains what staging means?
49. A peds nurse is taking history of a child 58. A 5 y.o male diagnosed with mild dehydration Extent of the disease at the time of diagnosis. 79. What statement is true concerning osteogenesis
hospitalized with Reye syndrome. Which of the should be given Oral Rehydration Solution (ORS) imperfecta (OI)? It is an inherited disorder.
following is a common occurrence seen in patients by the nurse or parent with which of the following 69. A child is admitted with acute
with Reye’s Syndrome? Cerebral Edema also rehydration therapy? 50 ml/kg/ 4 hrs glomerulonephritis. What should the nurse expect
fatty liver the urinalysis during this acute phase to show?
Hematuria and proteinuria.
80. A child with juvenile idiopathic arthritis (JIA) 90. An 18-month-old with pediculosis capitis (head 102. At which of the following ages is 112. Signs and symptoms of transfusion reaction?
is started on a nonsteroidal anti-inflammatory drug lice). What is the drug of choice? Permethrin 1%. recommendation made by the American Academy Chills, shaking, flank pain, hyperthermia (NOT
(NSAID). What nursing consideration should be of Pediatrics (AAP), for pre-adolescent and hypothermia).
included? Check for abdominal pain and bloody 91. The nurse is teaching the family of a child, age adolescent females to be screened for Scoliosis?
stools. 8-years-old, with moderate hemophilia about home 10-12. 113. Child with sickle cell crisis, which signs and
care. What should the nurse tell the family to do to symptoms shows a child is having minor cerebral
81. A child with juvenile idiopathic arthritis (JIA) minimize joint injury? Provide intravenous (IV) 103. The pediatric nurse is preparing to admit a 5- attack? Headache, weakness, visual
is started on a nonsteroidal anti-inflammatory drug infusion of factor VIII concentrates. year-old child with hepatitis A. Which of the disturbances.
(NSAID). What nursing consideration should be following is a clinical feature of hepatitis A nurses
included? Give NSAIDS between meals. 92. An 18-month-old child has been diagnosed with normally recognize? The mode of transmission is 114. In teaching, the parent of a newly diagnosed 2
pediculosis capitis (head lice). Which prescription primarily by the fecal oral route. y.o child with pyelonephritis related to
82. The nurse is discussing home care with a mother should the nurse question if ordered for the child? vesicoureteral reflux (VUR), the nurse should
whose 6-year-old child has hepatitis A. What Malathion (Ovide). 104. An 11-year-old girl with celiac disease was include which information? Have siblings
information should the nurse include? Teach discharged from the hospital. An appropriate examined for VUR
infection control measures to family members. 93. A 10-yar-old in peds clinic with fever, anorexia, teaching was carried out by the nurse if the parents
(Hepatitis A can be transmitted through the crampy abdominal pain preceding watery or bloody are aware of avoiding which of the following? 115. The nurse is assisting with application of a
fecal-oral route) diarrhea. What is the pathogen that causes the Wheat. synthetic cast on a child with a fractured humerus.
diarrhea? Shigella. What is an advantage of a synthetic cast over a
83. A pediatric nurse admits a 5-year-old with 105. The most common causative pathogen for plaster of Paris cast? Drying time is faster
Hepatitis A. Which of the following clinical 94. A 2-year-old child has a chronic history of diarrhea seen in children around the world is which
manifestation? Fecal oral transmission constipation and is brought to the clinic for of the following organisms? Rotavirus. 116. To assist in the prevention of urinary tract
evaluation. What should the therapeutic plan infections (UTIs) in children, which of the
84. A 3-month-old seen in peds clinic has seborrhea initially include? Bowel cleansing. 106. A pediatric nurse assesses a newborn with following is one of the best recommendations the
dermatitis (cradle cap). What instruction to give? symptoms of failure to pass meconium within 48 nurse should make to parents? Ensure clear
Shampoo the scalp daily until the crust is 95. The nurse is teaching infant care to parents with hrs after birth. Which of the following diseases will liquid intake of 2 L/day
removed. an infant who has been diagnosed with osteogenesis be suspected in this newborn? Hirschsprung.
imperfecta (OI). What should the nurse include in 117. The nurse notes that a child has lost 3.6 kg (8
85. The nurse is teaching parents about caring for the teaching session? “Lift the infant by the 107. A young girl has just injured her ankle at lb) after 4 days of hospitalization for acute
their infant with seborrheic dermatitis (cradle cap). buttocks, not the ankles, when changing school. In addition to notifying the child’s parents, glomerulonephritis. What is the most likely cause
Which statement by the parents indicates diapers.” what is the most appropriate, immediate action by of this weight loss? Reduction of edema
understanding of the teaching? “We will use a fine- the school nurse? Apply ice.
tooth comb to help remove the loosened crusts 96. What major complication is associated with a 118. A young girl has just injured her ankle at
from the strands of hair.” (antiseborrheic child with chronic renal failure? Water and 108. A 3-year-old child has a femoral shaft school. After notifying the child’s parents and
shampoo should be done daily). sodium retention. fracture. The nurse recognizes that the applying ice to the area, what is the next most
approximate healing time for this child is how appropriate action by the school nurse? Elevate
86. When giving instructions to a parent whose 97. What bone disorder of childhood is caused by long? 4 weeks. the extremity to reduce edema formation
child has scabies, what should the nurse include? Be an autosomal recessive defect in the synthesis of
prepared for symptoms to last 2 to 3 weeks. collagen? Osteogenesis imperfecta. 109. A 10 y.o male with Legg-Calve Perthes 119. The nurse is assessing an infant with
Disease is seen by a pediatrician. The student intussusception. Which of the following clinical
87. When giving instructions to a parent whose 98. A 4 yr old female seen in peds clinical has nurse is aware… Partial weight bearing on the findings should the nurse expect? Select all.
child has scabies, what should the nurse include? urinary symptoms. What is the medical terminology affected side, staging of the disease process, -Palpable sausage shaped abdominal mass
The scabies cream can be used for infants over 2 for inflammation of the bladder? Cystitis. traction. -Vomiting
months old. -Stool mixed with blood and mucous
99. A 3-day-old infant presents with abdominal 110. The nurse is preparing to admit a 7-year-old
88. In a child with sickle cell anemia (SCA), distention, is vomiting, and has not passed any child with Crohn disease. What clinical 120. The nurse is planning care for a school age
adequate hydration is essential to minimize sickling meconium stools. What disease should the nurse manifestations should the nurse expect to observe? child with bacterial meningitis. What intervention
and delay the vasoocclusion and hypoxia–ischemia suspect? Hirschsprung disease. Pain is common, diarrhea is moderate to severe, should be included? Assess for signs of increased
cycle. What information should the nurse share with weight loss is severe. intercranial pressure
parents in a teaching plan? Check for moist 100. A 4 week old infant has a femoral shaft
mucous membranes. fracture. The nurse recognizes that the approximate 111. Signs and symptoms of child in summer 121. The healthcare provider has prescribed
healing time for this infant’s fracture is how long? camp with Lyme disease? Treat with antibiotic sulfasalazine (Azulfidine) 5mg/kg PO/dose bid
89. In a child with sickle cell anemia (SCA), 2-3 weeks. Doxycycline, spirochete that enters skin with juvenile arthritis.The child weighs 55 lb. The
adequate hydration is essential to minimize sickling through tick bite, disease seen in three stages. nurse is preparing to administer the 0900 dose.
and delay the vasoocclusion and hypoxia–ischemia 101. A 1 week old infant is evaluated in pediatric Calculate the dose the nurse should administer in
cycle. What information should the nurse share with renal clinic with a narrowing of the opening of his mg? 125 mg
parents in a teaching plan? Monitor child for sign foreskin. What is this condition called? Phimosis.
of dehydration.
122. A healthcare provider prescribes ondansetron 131. The Transanal Soave pull through procedure 140. A child hospitalized with juvenile idiopathic 149. A 10 yr old female seen in a specialty peds
(Zofran) 0.15 mg/kg IV 30 minutes before is scheduled to be done on an infant in the PICU, arthritis (JIA) complains of pain in the knees. clinic has a diagnosis of Spina Bifida. On
chemotherapy for a child with acute lymphoblastic the peds nurse is aware that the procedure will What intervention should help to relieve the examination the peds nurse observes the spinal sac
leukemia. The child weighs 22 kg. The medication correct which of the following defect? discomfort? with meninges and nerves. Which of the following
label states: Ondansetron 2mg/1 ml. The nurse Aganglionic megacolon. Applying warm, moist compresses to the is the child’s diagnosis?
prepares to administer the dose. How many mL will knees. Myelomeningocele.
the nurse prepare to administer the dose? 132. The nurse is teaching the family of a child,
1.7 mL age 8 years, with moderate hemophilia about 141. A pediatric nurse is caring for a newborn in 150.During an absence seizure activity, what
home care. What should the nurse tell the family the NICU with clinical manifestations of bulging symptoms should the nurse expect the child to
123. A 3-day-old infant presents with abdominal to do to minimize joint injury? fontanel and distended scalp veins and separated experience?
distention, is vomiting, and has not passed any Provide intravenous (IV) infusion of factor sutures. Which of the following diagnosis the Brief loss of consciousness.
meconium stools. What disease should the nurse VIII concentrates. symptoms suggest? Hydrocephalus.
expect? 151. A 2-year-old has chronic constipation and is
Hirschsprung disease. 133. A 4 yr old in the PICU in unconscious after 142. A 5-year-old child is admitted to the hospital brought to the clinic for evaluation. The therapeutic
a MVA, and is at risk for increased incranial in a sickle cell crisis. Which of the following plan initially includes what?
124. A 10 yr. old child is seen in the peds clinic with pressure (ICP). What nursing intervention will symptoms would indicate the child is having a Bowel cleansing.
fever, colicky abdominal pain, and bloody diarrhea. reduce the risk of ICP? minor cerebral attack? Headache, Weakness
The pediatric nurse suspects the causative pathogen for Limit stimulation. 152. Staging is a part of the diagnostic evaluation of
this child’s diarrhea is which of the following 143. Which of the following treatment is the best a child with cancer. Which of the following explains
pathogens? 134. What intervention should be beneficial in method for a 7-month-old infant with a function of cancer staging?
Salmonella. reducing the risk of Reye syndrome? Developmental Dysplasia of the Hip (DDH)? Accurate staging is used as the basis for
Avoidance giving aspirin to children after a Closed Reduction with Spica Casting. treatment protocols.
125. While educating a new 7yr old diabetic patient suspected vital illness.
and the parents, the pediatric nurse includes which of 144. Which of the following statements best 153. The pediatric nurse is preparing to admit a
the following times during the day when the child’s 135. An infant with recurring atopic dermatitis describes diabetes insipidus? foreign born 4-year-old child with hepatitis A.
blood sugar level needs to be monitored? (eczema) is seen in peds clinic because of a recent Diabetes insipidus is characterized by the Which of the following is a clinical feature of
Check blood glucose levels ac and hs. exacerbation. Which suggestion should the nurse passage of large volumes of dilute urine. hepatitis A nurses normally recognize?
provide to the parents? The mode of transmission is primarily by the
126. The pediatric version of the Glasgow Coma Scale Apply petroleum jelly to skin after daily bath. 145. A pediatric nurse is taking care of a child fecal oral route.
is used on a 3-year-old male after falling from a tree in diagnosed with Pyelonephritis. The nurse is aware
his back yard. A score of 3 on his chart is associated 136. A 12 yr. old child at summer camp has a rash, that the most common cause of Pyelonephritis is 154. Accurate staging of which tumor is the
with? and the nurse suspects Lyme disease. Which of which of the following? mainstay of establishing initial treatment for a 12-
Deep coma. the following symptoms describe Lyme disease? VUR with infection. month-old infant with an abdominal mass that
(Select all that apply.) crosses the midline?
127. The nurse is preparing to admit a 7-year-old child Caused by a spirochete that enters the skin 146. How should a nurse instruct the parent of a Neuroblastoma.
with Crohn disease. What clinical manifestations through a tick bite. 3-month-old with seborrheic dermatitis (Cradle
should the nurse expect to observe? (Select all that Disease initially seen in any of 3 stages. Cap) to shampoo the child’s hair? 155. A 4 yr old male is evaluated in peds specialty
apply.) Treated with oral antibiotic Doxycycline. Use a soft brush or fine tooth comb to gently clinic with symptoms of muscular weakness,
Pain is common. scrub the area. lordosis, calf muscle hypertrophy, and frequent
Diarrhea is moderate to severe. 137. Which of the following are clinical falls. Which of the following is a likely diagnosis?
Anal and perianal lesions are common. manifestations of minimal change nephrotic 147. The x-rays of a 5 yr old child seen in peds Muscular dystrophy.
syndrome, usually seen in children with this clinic showed multiple fractures. The pediatric
128. A full-term male has hypospadias. Which disorder? nurse suspects child physical abuse. Which of the What type of dehydration is defined as
statement describes hypospadias? Massive proteinuria, hypoalbuminemia, and following diagnosis should also be considered in “dehydration that occurs in conditions in which
The urethral meatus opens on the underside of the edema. a child with multiple fractures electrolyte and water deficits are present in
penis. approximately balanced proportion?” Isotonic
138. A 4-year-old is seen in the ER with a fracture 148. Which nursing intervention is priority for the dehydration
129. Children with salt losing type of Congenital of the humerus. Which of the following P’s are pediatric intensive care nurse to implement when
Adrenal Hyperplasia (CAH) need which of the assessed to determine the extent of the child’s caring for a 15-yaer-old client diagnosed with
following replacement therapy? injury? diabetic ketoacidosis (DKA)?
Aldosterone. Pain and pulselessness. Assess for a fruity breath odor.

130. A child with cerebral palsy is seen in peds 139. Which of the following is a common
specialty clinic and will receive a Botox injection. The symptom seen in a newborn with a diagnosis of
peds nurse is aware the treatment is specifically for hypothyroidism?
which of the following conditions? Spasticity Constipation.
What do morbidity rates measure? The father of a hospitalized child tells the nurse, What statement is true concerning the increased When taking a child’s blood pressure, the nurse
Prevalence of a specific illness in a population “He can’t have meat. We are Buddhist and use of telephone triage by nurses? would select a cuff with a bladder width that is
vegetarians.” What is the nurse’s best Access to high-quality health care services has large enough to cover what percentage of the upper
What health promotion interventions have the intervention? increased. arm? 40%
greatest impact on injury prevention? Order the child a meatless tray as requested.
Using a developmental approach to safety What is most likely to encourage parents to talk What is the normal age for binocularity, the ability
counseling What statement is true concerning folk remedies about their feelings related to their child’s to fixate on one visual field with both eyes
in pediatric health care? illness? simultaneously, to be achieved?
What is the leading cause of death from unintentional They may be used to reinforce the treatment Use open-ended questions. 3 to 4 months
injuries in children? plan.
Motor vehicles What are some legal and ethical issues that arise The nurse has a 2-year-old boy sit in “tailor”
What genetic term refers to a recognized pattern for the nurse when using an interpreter? position while palpating for the presence of the
In 1935, Title V of the Social Security Act was of malformations with a single specific cause? The family should be fully informed of all testes. What is the rationale for this position?
passed. This was significant in the evolution of child Syndrome aspects of procedures before consenting. It prevents the cremasteric reflex.
health care in the United States because it established
what? What do trisomy 13, trisomy 18, and trisomy 21 What approach would be best to use to ensure a Kimberly is having a checkup before starting
Child Welfare Services have in common? receptive response from a toddler? kindergarten. The nurse asks her to do the “finger-
Diagnosis can be made early based on Focus communication on the child and tell to-nose test.” What is the purpose of this test?
What nursing intervention is most descriptive of physical characteristics. him or her how a procedure will feel. Cerebellar function
atraumatic care of children?
Preparing a child before any unfamiliar treatment What statement is accurate and describes the risk When the nurse interviews an adolescent, what is What is the most consistent and commonly used
or procedure of recurrence in autosomal dominant important to help establish a relationship? indicator of pain in infants?
inheritance? Display a genuine interest in the adolescent. Facial expression of discomfort
What is most suggestive that a nurse has a Each child of a heterozygous affected parent
nontherapeutic relationship with a patient or family? has a 50% chance of inheriting the mutated The nurse is taking a health history on a child. At The nurse is educating a new nurse on
Staff members are concerned about the nurse’s allele. the beginning of the interview, a parent says, “I identification of pain in children. What does the
actions with the patient or brought him here because he always has nurse teach about physiologic measurements in
family. A young woman who has recently become diarrhea.” Under what heading is this children’s pain assessment?
engaged to be married asks the nurse when information recorded? Not useful as the only indicator for pain
What is descriptive of nursing diagnoses? genetic counseling is advisable. The couple does Chief complaint
They provide the basis for the selection of nursing not plan to have children for several years. What What self-report pain rating scales can be used in
interventions. would be the nurse’s recommendation? What statement explains why it can be difficult children as young as 3 years of age?
Now if they are members of a population at to assess a child’s dietary intake? FACES Pain Rating Scale
risk for certain diseases Recall of food consumption is frequently
Which of the following terms best describes a group unreliable. What information does the nurse include when
of people who share a set of values, beliefs, practices, Elevation of triple marker screening results teaching parents about nonpharmacologic
social relationships, laws, politics, economics, and indicates the need for what during pregnancy? What approach is the most appropriate when strategies for pain management in children?
norms of behavior? Further diagnostic testing performing a physical assessment on a toddler? May reduce pain perception.
Culture Use minimum physical contact initially.
What blood or body-fluid is most often used in An intravenous line is needed in a school-age
.What is one of the top factors affecting children and chromosome analysis of a fetus? The nurse is assessing a 3-year-old African child. What medication is appropriate analgesic for
directedly related to health disparities? Amniotic fluid American child who is being seen in the clinic use with this patient?
Poverty for the first time. The child’s height and weight LMX (4% liposomal lidocaine cream) 30
A couple has given birth to their first child, a are at the 20th percentile on the commonly used minutes before the procedure
Maria is a Spanish-speaking 5-year-old girl who has boy with a recessive disorder. The genetic growth chart from the National Center for Health
started kindergarten in an English-speaking school. counselor tells them that the risk of recurrence is Statistics (NCHS). When interpreting these data, The nurses caring for a child are concerned about
Crying most of the time, she appears helpless and one in four for each child they have in the future. the nurse would recognize what? the child’s frequent requests for pain medication.
unable to function in this new situation. What is the What statement is a correct interpretation of this The NCHS charts are accurate for U.S. During a team conference, a new nurse suggests
best explanation for this behavior? information? African American children. they consider administering a placebo instead of
She is experiencing cultural shock. The risk factor remains the same for each the usual pain medication to see how the child
pregnancy. A 5 year old has patient-controlled analgesia responds. The team educates the nurse on why this
A child who has been receiving morphine (PCA) for pain management after abdominal is not appropriate and bases the decision on what
intravenously will now start receiving it orally. The surgery. What information does the nurse knowledge?
nurse would anticipate that to achieve equianalgesia include in teaching the parents about the PCA? This practice is unjustified and unethical.
(equal analgesic effect), the oral dose will be what The pump allows for a continuous basal rate
compared to the intravenous dose? Greater than the and delivers a constant amount of medication
intravenous dose to control pain.
The nurse is caring postoperatively for an 8-year-old What illnesses does respiratory hygiene and What tools would be most useful to assess An infant with hydrocephalus is hospitalized for
child with multiple fractures and other traumatic cough etiquette by the Centers for Disease maternal and newborn attachment behaviors? surgical placement of a ventriculoperitoneal shunt.
injuries from a motor vehicle crash. The child is Control and Prevention (CDC) prevent? NCAST (Nursing Child Assessment Satellite Postoperative nursing care would include what?
experiencing severe pain. What is an important RSV, influenza, and adenovirus Training) Feeding Scale Monitor closely for signs of infection.
consideration in managing the child’s pain? What clinical manifestations of developmental
Plan a preventive schedule of pain medication What is the most common piece of medical The nurse is assessing a 3-day-old breastfed dysplasia of the hip would be assessed in a
around the clock. equipment that can transmit harmful newborn who weighed 3400 g (7 pounds, 8 oz) newborn?
microorganisms among patients? at birth. The infant’s mother is now concerned Ortolani sign
What is a common significant side effect from opioid Stethoscope because the infant weighs 3147 g (6 pounds, 15
administration? Constipation oz). The most appropriate nursing intervention is A newborn with congenital clubfoot is being
A 12-month-old child presents to the clinic for a what? treated with serial casts. The parents ask why so
A school-age child with cancer is being prepared for well visit after missing several appointments. Explain that this weight loss is within normal many casts are required. What information is
a procedure. The child says, “I have had one of these The child began her immunization schedule but limits. accurate for the nurse to provide to the family?
before. They hurt.” The nurse bases her response on has missed several follow-up appointments and They allow for gradual stretching of tight
what knowledge related to pain in this patient? doses of immunizations. What is the most Where would nonpathologic cyanosis normally structures.
Commonly experience treatment-related moderate appropriate nursing intervention? be present in the newborn shortly after birth?
to severe pain when they have cancer The child should only receive the missed Feet and hands What is a major long-term problem for a child with
doses of immunizations based on catch-up a cleft lip and palate?
The mother of a child is hepatitis B surface antigen schedule. What findings on physical assessment of a Faulty dentition
(HBsAg) negative, when would receive his or her neonate would indicate the need for further
first dose of the hepatitis B virus (HBV) vaccine? What disease would require strict isolation of the evaluation? A newborn was admitted to the nursery with a
Birth before discharge from the hospital. patient? Low-set ears complete bilateral cleft lip and palate. The
Chickenpox physician explained the plan of therapy and
What is the appropriate site to administer an What findings would the nurse consider normal anticipates good results. However, the mother
intramuscular (IM) vaccine to a newborn? An important role of the nurse in ambulatory in assessing the anterior fontanel of a neonate? refuses to see or hold her baby. What would be
The vastus lateralis muscle settings and schools is the identification of Pulsating anterior fontanel included in the initial therapeutic approach to the
communicable diseases for treatment and the mother?
The hepatitis A vaccine is now recommended for the prevention of spread. What is an important The nurse is using a bulb syringe to suction a Encourage her to express her feelings.
first dose started at what age? component related to the first period of the neonate after delivery. What is an important
1 year contagiousness of disease? consideration? The nurse assesses a neonate after spitting up the
Prodromal stage Clear the pharynx before the nasal passages. first feeding and having a coughing episode during
What time frame has the Advisory Committee on the feeding. What assessment finding would
Immunization Practices (ACIP) of the Centers for What vaccinations are included in health In the newborn, into what muscle is indicate possibility of esophageal atresia or
Disease Control and Prevention (CDC) and American promotion during infancy? (Select all that intramuscular vitamin K administered? tracheoesophageal fistula?
College of Obstetricians and Gynecologists apply.) Vastus lateralis Excessive amount of frothy saliva in the mouth
recommended that pregnant adolescents and women Haemophilus influenzae type b (Hib)
who are not protected against pertussis receive the Diphtheria, tetanus, and pertussis (DTaP) What does the American Academy of Pediatrics What is the earliest clinical manifestation of biliary
tetanus, diphtheria, and pertussis (Tdap) vaccine? Poliovirus recommend as the best form of infant nutrition? atresia?
Between 27 and 36 weeks of gestation or Hepatitis B virus (HBV) Exclusive breastfeeding until 1 year of age Jaundice
postpartum before discharge from the hospital
The Hib conjugate vaccine protects an infant What is an important consideration in The nurse is caring for a neonate born with an
What is an important consideration when the nurse is against what diseases? (Select all that apply.) understanding the reactions of parents when their omphalocele. What does initial management of this
discussing enuresis with the parents of a young child? Bacterial meningitis infant is born with physical defects? neonate include?
The child should be encouraged to take charge of Epiglottitis The psychologic reaction is similar to that Covering the omphalocele with saline-soaked
treatment interventions. Bacterial pneumonia with the death of an infant. gauze and a plastic drape
Septic arthritis
A mother calls the school nurse saying that her Sepsis An infant is born with anencephaly. Based on the Hypospadias refers to what?
daughter has developed school phobia. She has been knowledge of this diagnosis, what information Urethral opening along ventral surface of penis
out of school for 3 days. What would the nurse What is the most critical physiologic change does the nurse consider when interaction with
include in recommendations to the mother? required of newborns at birth? the family? Mr. and Mrs. Wilson have a newborn with
Immediately return the child to school. Transition from fetal to neonatal breathing The condition is incompatible with life. ambiguous genitalia. Tests are being done to assist
in gender assignment. The parents tell the nurse
What assessment findings would be anticipated is a Neonates are predisposed to problems with What problem is most often associated with that family and friends are asking what caused the
child with depression? thermoregulation because of what factor? myelomeningocele? baby to be this way. What is the appropriate
Change in appetite, resulting in weight loss or A large body surface area favors heat loss to Hydrocephalus information for the nurse to provide?
gain the environment. Explain the disorder so they can explain it to
others.
What is the initial action in the emergency treatment What is characteristic of a neonate’s vision? Latex allergy is suspected in a child with spina What is important understanding for a nurse
of poisoning in a child? Pupils react to light. bifida. What are appropriate nursing working in an outpatient surgery center for
Assess the child. interventions to include in care of this patient? children? Families need to be prepared for what
A family wants to begin oral feeding of their 4- Avoid using any latex product. to expect after discharge.
A 7-year-old child has ingested a toxic dose of iron. year-old son, who is ventilator dependent and
The parent reports that the child vomited and had currently tube fed. They ask the home health What is a common clinical manifestation of What would the nurse consider when having
gastric pain an hour ago but “feels fine” now. The nurse to feed him the baby food orally. The autism? Early abnormal eye contact informed consent forms signed for surgery and
parent is not sure when the child ingested the iron nurse recognizes a high risk of aspiration and an procedures on children?
tablets. What is the appropriate recommendation by already compromised respiratory status. What is The nurse would make a referral for The risks and benefits of a procedure are part
the nurse? Bring the child to the hospital the most appropriate nursing action? communication impairment in what situations? of the consent process.
immediately. Acknowledge their request, explain the risks, First words not uttered before age 2 years
and explore with the family the available The nurse is planning how to prepare a 4-year-old
What is the most frequent source of symptomatic lead options. What represents the major stressor of child for a diagnostic procedure. What guidelines
poisoning in children? hospitalization for children from middle infancy are appropriate for preparing a preschooler?
Lead-based paint The home health nurse is planning care for a 3- throughout the preschool years? Tell the child procedures are never a form of
year-old boy who has Down syndrome and is on Separation anxiety punishment.
What is descriptive of a parent who is an abuser? continuous oxygen. He recently began walking
Is likely a single parent or from a young parent around furniture. He is spoon fed by his parents When a preschool child is hospitalized without The nurse is preparing a 12-year-old girl for a bone
family. and eats some finger foods. What is the most adequate preparation, how would the child likely marrow aspiration. She tells the nurse she wants
appropriate goal to promote normal view this hospitalization? Punishment her mother with her “like before.” What is the
What is a common characteristic of those who development? Encourage mobility. appropriate nursing action?
sexually abuse children? What is usually the greatest threat to a Grant her request.
Pressure victim into secrecy. What is the appropriate description of a young hospitalized adolescent?
child who has an intelligence quotient (IQ) of Fear of altered body image What would be helpful word(s) to substitute for the
Home care is being considered for a young child who 45? word “shot” when working with a 4-year-old
is ventilator dependent. What factor is most important Moderately cognitively impaired but What is a common initial reaction of parents to child? Medication under the skin
in deciding whether home care is appropriate? trainable illness or injury and hospitalization in their
Adequate family training child? Frustration When would clear liquids be stopped before a
When a child with mild cognitive impairment scheduled surgery? Two hours before surgery
For case management to be most effective, who is the reaches the end of adolescence, what A 6-year-old child needs to be hospitalized again
most appropriate case manager? characteristics would be expected? because of a chronic illness. The clinic nurse What is a potential cause of a postoperative
One nurse Achieves a mental age of 8 to 12 years. overhears her school-age siblings tell her that decrease in blood pressure?
they are “sick of Mom always sitting with her in Vasodilating anesthetic agents
The home health nurse is caring for a child who When would children with cognitive impairment the hospital and playing with her. . . . It isn’t fair
requires complex care. The family expresses be referred for stimulation and educational that you get everything and we have to stay with The nurse is caring for an unconscious 10-year-old
frustration related to obtaining accurate information programs? the neighbors.” What is the nurse’s best child. What would be included in skin care for this
about their child’s illness and its management. What As young as possible. assessment of this situation? child?
is the best action for the nurse? Jealousy and resentment are common Use a drawsheet to move the child in bed to
Answer questions in a straightforward manner Parents of a child with cognitive impairment ask reactions to the illness and hospitalization of a reduce friction and shearing injuries
and get professional assistance when an answer is the nurse for guidance in toilet training. They sibling.
unknown. have older children who were successfully toilet What is an appropriate intervention to encourage
When administering a gavage feeding to a school-age trained but do not know how to do this with the What is an appropriate nursing intervention to food and fluid intake in a hospitalized child?
child, what is the appropriate action by the nurse? impaired child. What would be included in the minimize separation anxiety in a hospitalized Give high-quality foods and snacks whenever
Position the child on the right side after nurses recommendation? toddler? Encourage parents to room-in. the child expresses hunger.
administering the feeding. Determine the child’s readiness to begin toilet
training. A 3-year-old child is being admitted for 1 week A 3-year-old child has a fever. Her mother calls the
What is defined as the forces that favor filtration from of hospitalization. Her parents tell the nurse that nurse reporting a fever of 38.8° C (102° F) even
the capillary? What is one of the major physical characteristics they are going to buy her “a lot of new toys though the child had acetaminophen 2 hr ago.
Capillary hydrostatic pressure and interstitial of a child with Down syndrome? because she will be in the hospital.” What What is the basis for the nurse’s actions?
oncotic pressure Hypotonic musculature understanding of the child’s age guides the Fevers such as this are common with viral
response by the nurse? illnesses.
What conditions would cause a decrease of fluid A child with Down syndrome may be screened At this age, children often need the comfort
requirements for children? for what before participating in some sports? and reassurance of familiar toys from home. A young child is brought to the emergency
Increased intracranial pressure Atlantoaxial instability department with severe dehydration secondary to
acute diarrhea and vomiting. What is the
therapeutic management of this child?
Intravenous fluids
Rapid replacement of fluid is essential in the Distortion of sound and problems in An 18-month-old child has just been admitted A 2-month-old breastfed infant is successfully
treatment of what types of dehydration? Hypotonic, discrimination are characteristic of what type of with croup. His parent is tearful and tells the rehydrated with oral rehydration solutions for acute
isotonic hearing loss? nurse, “This is all my fault. I should have taken diarrhea. What instructions are include to the
Sensorineural him to the doctor sooner so he wouldn’t have to mother specific to breastfeeding?
Depression of the central nervous system (CNS), be here.” What is appropriate in the care plan for Continue breastfeeding.
manifested by lethargy, delirium, stupor, and coma, is What assessment finding would suggest to the this parent?
observed in what? nurse a possible visual impairment in a child? Clarify misconception about the illness. The nurse is explaining to a parent how to care for
Metabolic and respiratory acidosis Excessive rubbing of the eyes a child with vomiting associated with a viral
A young child is diagnosed with vesicoureteral illness. What would the nurse include?
The nurse is assessing an infant brought to the clinic reflux. What is a common recurrent complication Brush teeth or rinse mouth after vomiting.
with diarrhea. He is lethargic and has dry mucous The nurse wears gloves during a dressing in a child with this diagnosis?
membranes. What would the nurse recognize as an change. What is the appropriate action when the Recurrent urinary tract infections The school nurse is called to the cafeteria because
early sign of dehydration? Tachycardia nurse removes the gloves? a child “has eaten something he is allergic to.” The
Wash hands thoroughly. What best describes the cause of most cases of child is in severe respiratory distress. What is the
What is an appropriate nursing action when caring for acute glomerulonephritis? priority nursing intervention?
a child with an intravenous infusion? A 2-year-old child is being admitted to the Antecedent streptococcal infection Have someone call for an ambulance or
Observe the insertion site frequently for signs of hospital for possible bacterial meningitis. What paramedic rescue squad.
infiltration. is the appropriate nursing intervention when In acute glomerulonephritis, what is the nurse is
preparing for a lumbar puncture? aware that is an early warning sign of What helps nurses understand how the respiratory
Several types of long-term central venous access Place the child in a side-lying position. encephalopathy? tract in children is different from the respiratory
devices are used. What is a benefit of using an Dizziness tract in adults?
implanted port (e.g., Port-a-Cath)? What is an important nursing intervention when Infants rely almost entirely on diaphragmatic-
Easy access for blood work performing a bladder catheterization on a young What are the common clinical manifestations of abdominal breathing.
boy? nephrotic syndrome?
What is a major complication of total parenteral Insert 2% lidocaine lubricant into the Proteinuria, hypoalbuminemia, and edema What is a measure of chest wall and lung
nutrition in children? Liver disease urethra. distensibility? Compliance
What is included in the therapeutic management
What would the nurse recognize as an early clinical The nurse needs to do a heel stick on an ill of nephrotic syndrome? What blood oxygenation tests is the photometric
sign of compensated shock in a child? neonate to obtain a blood sample. What is Corticosteroids measurement of oxygen saturation?
Apprehension recommended to facilitate this procedure? Oximetry
Wrap the foot in a warm washcloth. When does dialysis or transplantation becomes
What occurs in septic shock? Massive vasodilation necessary for a child in chronic renal failure? What is included in nursing considerations related
When giving liquid medication to a crying 10- Glomerular filtration rate falls below 10% to to the administration of oxygen in an infant?
What statement regarding burn injuries in children is month-old infant, what approach minimizes the 15% of normal concentration.
correct? The prognosis for a burned child is possibility of aspiration?
directly related to the amount of tissue destroyed. Administer the medication with an oral What is an advantage of continuous cycling The nurse is teaching a mother how to perform
syringe placed along the side of the infant’s peritoneal dialysis or continuous ambulatory chest physiotherapy and postural drainage on her
A child is developing respiratory failure. What are the tongue. peritoneal dialysis for adolescents who require 3-year-old child, who has cystic fibrosis. What
assessment findings to indicate the signs of hypoxia dialysis? would the nurse include in the instructions for
are becoming severe? Somnolence The nurse is caring for a 12-year-old boy who Adolescents can carry out procedures performing percussion?
sustained major burns when he put charcoal themselves. Cover the skin with a shirt or gown before
What is a primary intervention when an infant chokes lighter on a campfire. The nurse observes that he percussing.
on a piece of food? is “very brave” and appears to accept pain with What type of diarrhea is commonly seen in
Blows between the shoulder blades. little or no response. What is the most malabsorption syndromes because the intestine The nurse is caring for an infant with a
appropriate nursing action? cannot absorb nutrients or electrolytes? tracheostomy when accidental decannulation
The nurse is assessing a child with a cardiac history. Request a psychologic consultation. Chronic occurs. The nurse is unable to reinsert the tube.
The child’s extremities are cool with thready pulses, What would be the nurse’s next action?
and urinary output is diminished. What do the To prevent burns from hot water in the home, What is the viral pathogen that frequently causes Try inserting a smaller tube.
assessment findings suggest for this child? what is the recommended water heater acute diarrhea in young children?
Decreased contractility thermostat setting? Rotavirus
49° C (120° F) What is included in the assessment for the
What procedure use high-frequency sound waves A stool specimen from a child with diarrhea Glasgow Coma Scale?
obtained by a transducer to produce an image of What urine tests of renal function is used to shows the presence of neutrophils and red blood Eye opening and verbal and motor response
cardiac structures? Echocardiography estimate glomerular filtration? cells. This is most suggestive of what
Creatinine conditions?
Bacterial gastroenteritis
What would be included in nursing interventions for What factors predisposes the urinary tract to What is the initial therapeutic management of the The nurse is doing a neurologic assessment on a
a child after a cardiac catheterization? infection? child with acute diarrhea and dehydration? child whose level of consciousness has been
Assess the affected extremity for temperature and Short urethra in girls Oral rehydration solution variable since sustaining a cervical neck injury 12
color. The school nurse is discussing prevention of hr ago. What is the most essential in this
What is an important nursing responsibility acquired immunodeficiency syndrome with assessment? Reactivity of pupils
What is an early sign of heart failure that would be when a dysrhythmia is suspected? some adolescents. What is appropriate to
recognized by the nurse? Tachypnea Count the apical rate for 1 full minute and include? The nurse is caring for a child with multiple
compare with radial rate. Recreational drug users should not share injuries who is comatose. What information is
What would be included in nursing care of an infant needles or other equipment. accurate related to pain in this child?
with heart failure? The nurse is explaining blood components to an Requires astute nursing assessment and
Organize activities to allow for uninterrupted 8-year-old child. What is the nurse’s best What are included in nursing considerations management.
sleep. description and action of platelets? related to the administration of chemotherapeutic
Help your body stop bleeding by forming a drugs?
What heart defects causes hypoxemia and cyanosis clot (scab) over the hurt area. Many chemotherapeutic agents are vesicants
because desaturated venous blood is entering the What is the cause for the signs and symptoms that can cause severe cellular damage if drug The nurse is caring for a 2-year-old girl who is
systemic circulation? Tetralogy of Fallot when hemoglobin falls sufficiently to produce infiltrates. unconscious but stable after a car accident. Her
clinical manifestations? Tissue hypoxia parents are staying at the bedside most of the time.
What information would a nurse counseling parents What pediatric oncologic emergencies are What is an appropriate nursing intervention?
on the home care of the child with a cardiac defect The nurse suspects a child is having an adverse caused by the rapid release of intracellular Encourage the parents to hold, talk, and sing to
before corrective surgery include? reaction to a blood transfusion. What is the metabolites during the initial treatment of some her as they usually would.
Promote normality within the limits of the child’s initial action by the nurse? cancers? Acute tumor lysis syndrome
condition. Stop transfusion and maintain a patent Why are infants particularly vulnerable to
intravenous line with normal saline and new What are the most common signs and symptoms acceleration–deceleration head injuries?
What is an important nursing consideration when tubing. of leukemia related to bone marrow Musculoskeletal support of the head is
chest tubes will be removed from a child? involvement? Petechiae, fever, and fatigue insufficient.
Administer analgesics before procedure. The parent of a child receiving an iron
preparation tells the nurse that the child’s stools A child with lymphoma is receiving extensive What findings would indicate to the nurse further
What is included in the therapeutic management of are a tarry green color. What information does radiotherapy. What is the most common side assessment and treatment is needed for a child with
the child with rheumatic fever? the nurse included in discussion with this parent? effect of this treatment? Malaise mild head injury? Confusion or abnormal
Administration of penicillin Normally expected change resulting from the behavior
iron preparation What is included in the postoperative care of a
What is the primary therapy for secondary preschool child who has had a brain tumor A 3-year-old child is hospitalized after submersion
hypertension in children? What is a priority nursing consideration when removed? Carefully monitor fluids because of injury. The child’s mother complains to the nurse,
Treatment of underlying cause caring for a child with sickle cell anemia? cerebral edema. “This seems unnecessary when he is perfectly
Teach the parents and child how to recognize fine.” What is the appropriate response by the
What statement best describes hypopituitarism? the signs and symptoms of crises. What best describes a neuroblastoma? nurse?
Skeletal proportions are normal for age. The diagnosis is usually made after metastasis “The reason for hospitalization is that
The parents of a child hospitalized with sickle occurs. complications could still occur.”
What would the nurse include when discussing a cell anemia tell the nurse that they are concerned
child’s precocious puberty with the parents? about narcotic analgesics causing addiction. The What is most descriptive of the therapeutic The nurse is admitting a young child to the hospital
Dress and activities should be appropriate to nurse would explain what concerning narcotic management of osteogenic sarcoma? because bacterial meningitis is suspected. What is
chronologic age. analgesics? Treatment usually consists of surgery and a priority of nursing care?
When they are medically indicated, children chemotherapy. Administer antibiotic therapy as soon as it is
What is the primary clinical manifestation of diabetes rarely become addicted. available.
insipidus? Where are Wilms tumors (nephroblastomas)
Polyuria, polydipsia Chelation therapy is begun on a child with alpha- located? Kidney A child is brought to the emergency department
thalassemia major. What is the purpose of this after experiencing a seizure at school. He has no
A 13-year-old girl is brought to the clinic with the therapy? Eliminate excess iron. What is a clinical manifestation of increased history of seizures. The father tells the nurse that
complaint of insomnia and hyperactivity. Other intracranial pressure (ICP) in infants? he cannot believe the child has epilepsy. What is
symptoms include gradual weight loss despite a good The school nurse is caring for a child with Irritability the best response by the nurse?
appetite; warm, flushed, and moist skin; and hemophilia who fell on his arm during recess. “The seizure may or may not mean that your
unusually fine hair. What are these assessment What supportive measures would the nurse What statement is correct regarding sports child has epilepsy.”
findings suggestive of for this child? implement first? injuries during adolescence?
Hyperthyroidism Elevate the arm above the level of the heart. Adolescents may not possess the insight and
judgment to recognize when an activity is
What factors promote(s) wound healing? beyond their capabilities.
Moist, crust-free wound environment
The nurse is caring for a child hospitalized with acute What is an important nursing consideration when What results when ice is applied immediately A hospitalized child with minimal change
adrenocortical insufficiency. Because of the sudden, caring for a child with impetigo contagiosa? after a soft tissue injury, such as a sprained nephrotic syndrome is receiving high dose
severe nature of the disease, the family needs a great Thoroughly wash hands and maintain ankle? prednisone. What nursing goal is appropriate for
deal of emotional support. What is the most cleanliness when caring for an infected child. Produces deep tissue vasodilation. this child?
appropriate nursing action? Promote adherence to the antibiotic regimen
Prepare the family for each procedure. What is the cause of ringworm, a skin disorder A 4-year-old child is newly diagnosed with
frequently found in schoolchildren? Legg-Calvé-Perthes disease. Nursing A 4 yr old with Spina Bifida is prepared for a
What statement best describes Cushing syndrome? Fungus considerations include what? straight catherization by the peds nurse. Which of
It is caused by excessive production of cortisol. Teach the family the care and management of the following actions by the nurse is recommended
What would the nurse include when explaining the corrective appliance. for this child?
The parent of a 10-year-old child with diabetes asks how to manage pediculosis capitis? Medicate the child with pain meds before the
the nurse why home blood glucose monitoring is Remove nits with a fine-tooth comb or The nurse is preparing an adolescent girl for procedure
being recommended. What is the basis for the nurse’s tweezers. surgery to treat scoliosis. What would the nurse
explanation? Children have a greater sense of include? Blood administration may be an Which of the following pain tool is most
control over the diabetes. Matt’s mother tells the nurse that he keeps option. appropriate for use in a 10 yr old child with Sickle
scratching the areas where he has poison ivy. Cell Anemia, to outline the area of the child’s
A school-age child recently diagnosed with type 1 What is the basis of knowledge for the response What is an important nursing consideration when pain?
diabetes mellitus asks the nurse if he can still play by the nurse? caring for a child with juvenile idiopathic APPT scale
soccer, play baseball, and swim. What is the basis of Scratching the lesions may cause them to arthritis? Teach the child and family the
knowledge for the response by the nurse? become secondarily infected. correct administration of medications. Which of the following should be included in the
Exercise is not restricted unless indicated by other nursing care of a child with autism spectrum
health conditions. The nurse stops to assist a child who has been hit A 6-month-old infant does not smile, has poor disorder (ASD)
by a car while riding a bicycle. Someone has head control, has a persistent Moro reflex, and Assign the child to a private room
When a child develops diabetic ketoacidosis, what is activated the emergency medical system. Until often gags and chokes while eating. What are
this considered by the nurse? paramedics arrive, the nurse would consider these findings are most suggestive of in this The nurse provides feeding instructions to a parent
A life-threatening situation what in caring for this child who has experienced infant? Cerebral palsy of an infant diagnosed with gastro-esophageal
severe trauma? reflux (GER). Which instruction should the nurse
When determining the etiology of a skin problem, Rapid assessment should begin with ABC The parents of a child with cerebral palsy ask the give the parent to assist in reducing the episodes of
what is essential to the diagnosis of the lesions? status: airway, breathing, and circulation. nurse if any drugs can decrease their child’s emesis? (Select all that apply)
Careful inspection spasticity. What is the basis for the nurse’s Provide smaller more frequent meals
Immobilization causes what effects on the response? Implantation of a pump to deliver Thicken the feedings by adding rice cereal to
What is the appropriate term to describe an abnormal cardiovascular system? Venous stasis medication into the intrathecal space the formula
sensation such as burning or prickling? decreases spasticity. Burp the infant frequently during feeding
Paresthesia A young child has recently been fitted with a
knee, ankle, and foot orthosis (brace). What What clinical manifestations in an infant would Which of the following statements best describes
After surgical repair of a fractured femur a 10 yr old would be included in care of the skin? be suggestive of spinal muscular atrophy Duchenne (pseudo-hypertrophic) muscular
has a long leg cast on. The toes on the child’s cast leg Contact a practitioner or orthotist if skin (Werdnig-Hoffmann disease)? dystrophy (DMD)?
are edematous, but has sensitivity, and movement. redness does not disappear. Lying in the frog position It had an X-linked inheritance problem
What action should the nurse take? It is characterized by presence of the Gower
Elevate the foot and leg on a pillow A child, age 10 years, sustained a fracture in the What is included in the therapeutic management sign
epiphyseal plate of her right fibula when she fell of a child with tetanus? Child exhibits a waddling gait, and lordosis
Evidence-based practice in clinical management is out of a tree. What does the nurse consider when Antibiotics to control bacterial proliferation
grounded in which of the following? discussing this injury with her parents? at the site of injury The nurse is doing a pre-hospitalization orientation
Clinical Research Studies Bone growth can be affected by this type of for a girl, age 7 years, who is scheduled for cardiac
fracture. An adolescent has sustained a spinal cord injury. surgery. As part of the preparation, the nurse the
A preschool child needs a dressing change to an area What are the characteristics of the first stage, nurse explains that after the surgery, the child will
with a 2nd degree thickness burn. To prepare the What would cause a nurse to suspect that an known as spinal shock syndrome? be in the intensive care unit. How might the
child, what strategy should the nurse implement? infection has developed under a cast? Flaccid paralysis below level of damage explanation by the nurse be viewed?
Administer pain medication to prep for the “Hot spots” felt on the cast surface Necessary information for the child and parents
procedure A young boy has just been diagnosed with
A nurse is reviewing the laboratory values of a pseudohypertrophic (Duchenne) muscular An 15 month old child seen in the Peds clinic is
child with rheumatic heart disease. Which dystrophy. What is included in his plan of care diagnosed with Hypertonic Dehydration. Which of
The nurse is preparing to administer a unit of packed finding does the nurse conclude is related to this for his family? the following is true of Hypertonic Dehydration?
red blood cells to a hospitalized child. What are condition? Recommend genetic counseling. Electrolyte deficits is less than water deficits
appropriate actions that applies to administering Positive antistreptolysin titer
blood?
Administer the blood with normal saline in a A disorder in which the basement membrane of The components of the FLACC scale include A 9 yr old seen in pediatric endocrine clinic has
piggyback setup the glomeruli becomes permeable to plasma cry, leg movement, facial expression, and symptoms of Cushing’s syndrome. What is the
Administer the first 50ml of blood slowly and stay proteins describes which of the following activity. Choose the missing component from the most common cause of Cushing’s syndrome as
with the child disorders? following list. manifested in children?
Administer blood through an appropriate filter Nephrotic syndrome Consolability Adrenal adenoma

Which of the following is the most common Which of he following toys are appropriate for a The nurse is caring for an adolescent client The health care provider has prescribed
symptom of Seborrhea Dermatitis? hospitalized preschool age child? receiving intravenous (IV) morphine for severe ondansetron (Zofran) 0.1 mg/kg as needed for a
Scalp rash Coloring books, building blocks, clay pain. The nurse observes a respiratory rate child admitted for vomiting. The child weighs
shallow, and the client cannot be aroused. What 55lbs. Calculate the correct dose of Zofran in
A 10 yr old male with Legg-Calve Perthes Disease is A newborn female infant in the NICU has signs priority nursing action should the nurse take? milligrams.
seen by the pediatrician. The student nurse is aware of ambiguous genitalia, and hyponatremia. Discontinue intravenous infusion 2.5
that management of this disease includes which of the Which of the following is the infant’s likely
following? diagnosis? A parent of a child with a disability expresses The nurse is caring for an 18-month-old with
Rest, staging of the disease process, traction Congenital Adrenal Hyperplasia feelings of guilt to the nurse about the child’s salmonella. Which of the following clinical
condition. What nursing intervention is most manifestations would the nurse expect to observe?
Which clinical finding may be present in an older Major goals for the therapeutic management of appropriate in this situation? Colicky abnormal pain and bloody diarrhea
child with Coarctation of the Aorta? juvenile idiopathic arthritis (JIA) include? Explore the parent’s adjustment responses
Diminished pulse in the lower extremities Control pain, preserve joint function The nurse is teaching the family of a child with
Prevent skin breakdown, relieve symptoms Type 1 diabetes about insulin. What should the
A school-age child with cystic fibrosis takes four Reduce joint discomfort, regain proper nurse include in the teaching session?
enzyme capsules with meals. The child is having one alignment Diabetic supplies should not be left in a hot
to two bowel movements per day. The nurse action in environment
regard to the pancreatic enzyme is based on the After studying about seizures, the student nurse After it has been opened, insulin is good for
knowledge that the dosage is what? understands which of the following? up to 28 to 30 days
Needs to be increased to decrease the number of Complex partial seizures result in no loss of Insulin bottles that have been opened should
bowel movements per day consciousness be stored at room temp or refrigerated

Which of the following is a common symptom seen The health care provider has prescribed Which assessments are included in the Apgar
in an infant diagnosis of hypothyroidism? famotidine (Pepcid) 1mg/kg/day divided BID for scoring system?
Lethargy a child with gastroesophageal reflux disease, Muscle tone, color, reflex irritability
child weights 33lb. The nurse is preparing to
administer the 0900 dose. Calculate the dose the The healthcare provider has prescribed
nurse should administer in milligrams. sulfasalazine (Azulfidine) 5mg/kg PO per dose
7.5 twice a day for a child with juvenile arthritis.
The child weighs 55lb. The nurse is preparing to
The health care provider prescribes vancomycin administer the 0900 dose. Calculate the dose the
200mg per intravenous piggy back (IVPB) every nurse should administer in milligrams.
6 hours for a child with cystic fibrosis. The 125mg
pharmacy sends the medication to the unit in a
150ml bag with directions to run the medication The health care provider had prescribed
over 120 minutes. What milliliters per hour will diazepam (Valium) 0.8mg/kg/day PO for a child
the nurse set the intravenous pump to run the with cerebral palsy who weighs 110lb. The nurse
medication over 120 minutes? is preparing to administer the 1200 dose.
75ml Calculate the dose the nurse should administer in
milligrams
What information should the nurse include when 10mg
teaching an adolescent with inflammatory Bowel
disease (IBD)? One of the most important interventions when
How to cope with stress and adjust to chronic caring for an infant with myelomeningocele in
illness the preoperative stage is which?
Watch for sign that might indicate developing
hydrocephalus
1. A 1-year-old child goes to day care Give tepid baths with mild soap Monitor for a gallop heart rhythm and Promote a quiet period upon waking in the
is recovering from an episode of Keep child's nails well-trimmed decreased urine output. morning
otitis media. Which intervention is Swaddle the infant during procedures
most important for the nurse to 2. What is the priority when caring for a 6- 2) A nurse is assessing a 1-month-old infant with an
recommend to the patients in order to month-old diagnosed with atopic atrial septal defect (ASD). Which assessment 7) A nurse is assisting a new mother as she is
prevent recurrence? dermatitis? finding does the nurse expect? breastfeeding her infant. The infant has been
Ans: Smoking cessation by the Ans: Prevention of scratching Murmur diagnosed with tetralogy of Fallot. During feeding,
parents the infant becomes cyanotic and is having difficulty
3. The parent of a 6-year-old calls the nurse 3) An 8-month-old infant is scheduled for a breathing. What should be the nurse's first action?
2. A nurse is planning to test the visual and reports that the child was playing femorally inserted balloon angioplasty of a Place infant in knee to chest position
acuity of a 7-year-old. Which is the outside in the snow and the child's feet congenital pulmonic stenosis in the cardiac
best way to test visual acuity in this now appear red and swollen. What is the catheterization laboratory. Which finding should the 8) The nurse is providing discharge instructions to
child? best response by the nurse? Ans: Place nurse report to the health care provider that could the parent of a child with Kawasaki disease. The
Ans: Position the child at a the child's feet in warm water possibly delay the procedure? nurse informs the parent that the presence of which
distance of 10 ft (3 m) from a immediately Infant has severe diaper rash symptom should be immediately reported to the
chart. health care provider?
4. The nurse reinforces teaching for the 4) The nurse receives report on 4 pediatric clients Fever
3. Which assessment findings should parents of a child with impetigo. Which of on a telemetry unit. Which client should the nurse
the nurse anticipate in a child with the following statements by a parent assess first? 9) A 3-month-old client has stopped breathing.
suspected acute otitis media (AOM)? indicates correct understanding of Infant client with ventricular septal defect with Identify the area where the nurse should check the
Select all that apply. teaching? Select all that apply. reported grunting during feeding client's pulse.
Frequent pulling on the affected Ans: The brachial artery is used to detect a pulse in an
ear, Refusal to eat, Restlessness 5) The emergency nurse is admitting a 12-year-old unresponsive client age <1 year.Located in
and irritability “I should wash my hands before and after client who reports palpitation. Which action should between armpit and elbow.
touching the infected area.” the nurse anticipate?
4. The health care provider (HCP) “I will separate my child's towels from other Instructing the client to hold their breath and 10) The nurse is caring for a newborn with patent
prescribes a 10-day course of amoxicillin for a 1- laundry and wash with hot water.” bear down ductus arteriosus. Which assessment finding should
year-old diagnosed with acute otitis media (AOM). “My child's fingernails should be kept short and the nurse expect?
Which instruction is most important for the nurse to well-filed.” Click the exhibit button for additional Loud machine-like murmur.
review with the child's parents? “The infection could easily spread to other information
Ans: Return to the office if the child does not children who come in contact with my child.” VS 11) The nurse is assisting with an education
improve within 48-72 hours Temp – 97.1 conference for graduate nurses about infant CPR.
B/P – 114/74 Which of the following statements are appropriate
5. A child's arm is burned from accidentally
5. A 2-year-old is suspected of having HR – 234 to include in the teaching? Select all that apply.
spilling boiling water on it, and the parent RR – 24
retinoblastoma. The nurse recognizes which sign as “A single rescuer responding to an unwitnessed
being most characteristics of this disease? calls the clinic. The nearest emergency SpO2 – 97% infant arrest should perform 2 minutes of CPR
department is an hour away. Which
Ans: Absent of red reflex before retrieving a defibrillator.”
instructions would be appropriate to give Physical Assessment “You should assess the infant’s brachial pulse
the parent? Select all that apply.
6. The nurse is performing an assessment on Neuro – Alert and oriented to person, place, and for no longer than 10 seconds.”
Ans:
a 2-year-old with otitis media. Which of the time. Follows commands
following actions would be appropriate? Select all “Briefly soak the arm with cool water.” Cardio – Tachycardia noted. + 1 radial pulses. 12) When monitoring an infant with a left-to-right-
that apply. “Cover the area with a clean, dry cloth.” +1 carotid pulses. Jugular veins 1 cm at 45- sided heart shunt, which findings would the nurse
Ans: Do not insert the speculum into the bony “Remove clothing, if not stuck to skin, degree angle. expect during the physical assessment?
inferior part of the ear canal around the burn.” Resp – Clear to auscultation. No adventitious Diaphoresis during feedings
Encourage the parents to have the child sounds. Even and unlabored breathing. Heart murmur
vaccinated against influenza and pneumonia Skin – Color appropriate for ethnicity. No Poor weight gain
Inspect the tympanic membrane for 6. The clinic nurse is reviewing self-care cyanosis or mottling.
redness, bulging, and perforation management of acne vulgaris with an Extremities – No edema in upper or lower 13) Click on the area where the nurse should place
Wait until the end of the assessment to adolescent client. Which client statement extremities. the thumbs or fingers to provide chest compressions
perform the otoscopic examination indicates a need for further instruction? for an infant.
Ans: “I have been scrubbing my face 6) A nurse is teaching the parents of an infant with The thumbs should be positioned side by side,
1. A 2-year-old is diagnosed with atopic twice daily with antibacterial soap.” tetralogy of Fallot. Which of the following actions just below the nipple line
dermatitis (eczema). Which instructions should the nurse include to reduce the incidence of
should the nurse teach the parents? Select 1) The nurse is planning care for a child being hypercyanotic spells? Select all that apply. 14) A nurse auscultates a loud cardiac murmur on a
all that apply. admitted with Kawasaki disease and should give Encourage smaller, frequent feedings newborn with suspected trisomy 21 (Down
Ans: Apply emollient immediately after priority to which nursing intervention? Offer a pacifier when the infant begins to cry syndrome). A genetic screen and an
bath. echocardiogram are scheduled that day. The
neonate's vital signs are shown in the exhibit. What Use of saline drops and a bulb syringe 4-month-old who has sclera visible 170/80 mm Hg, and proteinuria. When
would be an appropriate action for the nurse to to suction nares. above the iris (sunset eyes). developing a plan of care for this client,
complete next? the nurse should include which most
Document the assessment finding 6. A 12-month-old is found to have a 12. The public health nurse conducts a accurate indicator of fluid loss or gain?
moderately elevated blood lead level. teaching program for parents of infants. Daily weight measurements.
VS Which of the following is the most Which statement by a participant
Temp – 98.6 serious concern for this child? indicates that teaching has been 19. A 3-month-old who weighs 8.8 lb (4 kg)
HR – 146 Neurocognitive impairment. successful? has just returned to the intensive care unit
RR – 42 “My infant should be able to pick up after surgical repair of a congenital heart
SpO2 – 98% 7. The nurse is providing teaching for small finger foods by age 10 months.” defect. Which finding by the nurse should
parents of a child diagnosed with fifth be reported immediately to the health care
1. The nurse is reinforcing education to the disease. Which statement by a parent 13. The nurse is teaching the parents of a 4- provider?
caregivers of a 9-year-old client indicates a need for further teaching? month-old who has developed positional Chest tube output of 50 mL in the past
diagnosed with scarlet fever. The client “Our child’s condition is plagiocephaly (flat head syndrome). hour.
has a history of type 1 diabetes mellitus. communicable until the rash Which statement by the parents indicates
Which statement by the caregivers disappears.” a need for further teaching? 20. The nurse is performing well-child
indicates that further teaching is needed? “I should place the infant in the prone examination in a pediatric clinic. Which
“We will not administer insulin if our 8. The nurse is planning care for an 8-year- position during naps.” finding requires further evaluation?
child is unable to eat.” old client with mild cognitive impairment Lateral curvature to the spine noted on
who is hospitalized for diagnostic testing. 14. The parent of a 5-year-old child calls the examination of a 10-year-old girl.
2. The nurse receives new prescription for a Which of the following interventions are clinic to report the recurrence of a
6-month-old client with bacterial appropriate to include in the plan of care? nosebleed for which the child was seen a 21. The nurse assists with a staff education
meningitis. Which action is the priority of (Select all that apply.) week ago. Which of the following conference about appropriate
care? Consistently assign the same nurse and instructions should the nurse reinforce? nonpharmacological pain-management
Administer 400 mg ceftriaxone IV unlicensed assistive personnel to care (Select all that apply.) interventions for newborns and infants.
every 12 hours. for the client. Apply a cold cloth to the bridge of the Which of the following strategies should
Provide appropriate toys based on nose. be included in the presentation? (Select
3. The nurse plans care for a child admitted developmental level rather than Apply pressure by pinching the nostrils all that apply.)
with measles. Which of the following chronological age. together. Administer an oral sucrose solution to a
interventions will the nurse include in the Reinforce parental limit-setting Attempt to keep the child calm and newborn during a circumcision
plan of care? (Select all that apply.) measures for preventing self-injurious quiet. procedure.
Advise measles vaccination for behavior. Assist the parent to hold a newborn
susceptible family members. Use a picture board to facilitate 15. What is the best activity for a school-aged skin-to-skin during an immunization
Place the client in a negative-pressure communication and promote child hospitalized for vasoocclusive sickle injection.
isolation room. understanding of procedures. cell crisis? Offer a pacifier to an infant while
Use a N95 respirator mask during Watching a favorite movie. performing venipuncture.
client contact. 9. The nurse receives 4 prescriptions for a Swaddle an infant while leaving one
child diagnosed with hemophilia A who 16. The nurse is caring for an infant with arm unwrapped during an IV dressing
4. A 12-month-old client has a high blood was brought to the emergency department suspected meningitis and preparing to change.
lead level of 18 mcg/dL. The nurse following an injury on the school assist with a lumbar puncture. What is the
educates the parents about lead poisoning. playground. The child has vomited once appropriate nursing intervention? 22. The nurse is reviewing discharge
Which statements made by the parent and has a headache. Which prescription Hold the child with the head and knees instructions with the parents of a child
indicate that teaching is successful? should the nurse carry out first? tucked in and the back rounded out. who just had a tracheostomy. Which
(Select all that apply.) Administer IV factor VIII. statement made by the parents indicates
“I should get our home inspected for 17. The parent of an 11-month-old child calls teaching has been effective?
the source of lead.” 10. Several 12-month-old infants are brought the pediatric outpatient clinic and tells the “I will always travel with two
“I will wash my child’s hands often, to the clinic for routine immunizations. nurse that the child was exposed to tracheostomy tubes, one of the same
especially before eating.” Which situation would be most important measles 2 days ago during a family trip to size and one a size smaller.”
“We will have to return for a follow-up for the nurse to clarify with the provider a theme park. What is the best response
lead level.” before administering the vaccination? by the nurse? 23. Which of the following statements made
Varicella-zoster vaccine for client Bring the baby into the clinic for the by the mother of a child recently
5. The nurse in a clinic is caring for an 8- recently diagnosed with leukemia. measles, mumps, rubella (MMR) diagnosed with celiac disease indicates a
month-old with a new diagnosis of vaccine. need for further teaching?
bronchiolitis due to respiratory syncytial 11. The clinic nurse performs assessments on “My child can have small amounts of
virus (RSV). Which instructions can the four infants. The nurse should alert the 18. A client diagnosed with acute foods containing wheat as long as she
nurse anticipate reviewing with the health care provider to see which client glomerulonephritis has pitting edema in remains symptoms free.”
parent? first? both lower extremities, blood pressure of
24. The nurse is reviewing discharge teaching “My parents always tell me that I am 36. The nurse is caring for a 4-year-old who Encourage the client to have peers visit
with the parent of a child with acute sitting too close to the television.” was hospitalized with influenza. Which while hospitalized.
myelogenous leukemia who was admitted nursing action would be most effective to Include the client as an active
with varicella-zoster virus. The client has 30. The nurse is caring for a child who has maintain psychosocial integrity? participant when planning care.
multiple lesions that have not crusted. had a tonsillectomy. Which of the Providing crayons to draw noses on Support the client in discussing
Which of the following instructions following are appropriate nursing facemasks. concerns about body image changes.
should the nurse include? (Select all that interventions? (Select all that apply.)
apply.) Anticipate ear pain and give 37. The parents of a hospitalized preschooler 43. The nurse is admitting an infant who has
“Apply diphenhydramine cream acetaminophen as needed. are concerned because their toilet-trained severe growth deficiency and facial
sparingly to lesions after bathing your Educate parents to expect the child to child has started welting the bed. Which characteristics of indistinct philtrum, a
child.” develop bad breath postoperatively. response by the nurse is most helpful? thin upper lip, and short palpebral
“Keep giving your child the acyclovir at Notify the health care provider about “It is normal for your child to regress fissures. Which question should the nurse
home as prescribed to fight the virus.” frequent, increased swallowing. while hospitalized.” ask to assess the cause of these clinical
“You can give your child findings?
acetaminophen for pain and fever.” 31. The nurse is gathering data on a 5-week- 38. The nurse is caring for an infant “Is there a history of exposure to
“Your child will no longer be infectious old admitted with a suspected diagnosis of diagnosed with Hirschsprung disease who alcohol in utero?”
after all the lesions crusted over.” pyloric stenosis. The nurse should expect is awaiting surgery. Which assessment
to find which laboratory value? finding requires the nurse’s immediate 44. The community health nurse is planning
25. The nurse is discussing feeding and eating Hematocrit of 57% (0.57) action? an education program for adolescents
practices with the mother of a 1-year-old. Episode of foul-smelling diarrhea and about skin care for acne vulgaris. Which
Which statement made by the mother 32. The nurse is caring for a 2-year-old who fever. of the following teaching points are
indicates a need for further instruction? is receiving a saline enema for treatment appropriate to include? (Select all that
“My child is fussy at bedtime so I put of intussusception. Reporting which client 39. What socioeconomic indicators would the apply.)
him to sleep with a bottle of milk.” finding to the health care provider (HCP) nurse identify as risk factors for a 2- A well-balanced diet can help support
is most important? month-old infant to develop failure to healthy skin.
26. The parent of a 21-day-old male infant Passed a normal brown stool. thrive (FTT)? (Select all that apply.) Antibacterial soap is harsh and can
reports that the infant is “throwing up a Infant lives only with mother, who is worsen acne.
lot.” Which assessment should the nurse 33. The nurse is caring for a 7-year-old client currently unemployed. Squeezing or picking lesions may
make to help determine if pyloric stenosis diagnosed with nephrotic syndrome who Infant’s primary caregiver has increase the risk for infection and
is an issue? (Select all that apply.) will be discharged soon. Which statement cognitive disabilities. scarring.
Assess the parent’s feeding technique. by the parent indicates the need for Parents are socially and emotionally Use skincare products labeled as
Check if the vomiting is projectile. further teaching? isolated. noncomedogenic to prevent clogging
Compare current weight to birth “I’ll organize playdates to keep my skin pores.
weight. child’s spirits up during relapses.” 40. A 2-month-old infant has been admitted
to the hospital with suspected shaken 45. A 2-year-old in the emergency
27. A 15-month-old begins to seize during 34. The nurse is caring for an infant with baby syndrome (abusive head trauma). In department is suspected of having
assessment for a high-grade fever. What osteogenesis imperfecta admitted with a reviewing the infant’s chart, the nurse intussusception. Which assessment
is the most appropriate nursing action? new fracture. The client also has old expects to encounter which of these finding should the nurse expect?
Stay with the client and monitor oxygen fractures in multiple stages of healing but clinical findings? Stools mixed with blood and mucus.
saturation levels. no bruising, abrasions, or redness of the Irritability and vomiting.
skin. What nursing intervention should be 46. The nurse is caring for a 4-year-old client
28. A nurse on a pediatric unit is admitting a included in the plan of care? 41. The mother of a 6-year-old child with with cystic fibrosis who uses a high-
school-aged child with suspected Reye Obtain blood pressure manually to cystic fibrosis (CF) has received frequency chest wall oscillation
syndrome. Which information obtained avoid cuff over-tightening. instruction on the use of pancreatic (HFCWO) vest for chest psychotherapy.
during the history taking is most enzymes. Which statement made by the After reinforcing education with the
consistent with this condition? 35. A mother reports to the pediatric nurse mother indicates a need for further client’s parents, which statement by a
Recent influenza infection. that her 3-year-old child coughs at night teaching? parent requires further teaching?
and at times until he vomits. The “It is okay for my child to chew this “I will allow my child to have a snack
29. The nurse is performing visual acuity symptoms have not improved over the medication.” while using the HFCWO vest to
screenings on a group of students. Which past 2 months despite multiple over-the- encourage cooperation.”
of the following student comments does counter cough medications. What should 42. The pediatric nurse cares for a 16-year-
the nurse recognizes as indicating the nurse explore related to a possible old client who is scheduled for an 47. The clinic nurse is caring for a 3-year-old
possible myopia? etiology? appendectomy in the morning. Which of client. Which task, if not observed or
“I have to hold my book close to my Ask about exposure to triggers such as the following interventions are reported by the parents as accomplished,
face so that the words are clear.” pet dander. appropriate to support the client’s will cause the nurse concern?
“If I squint or close one eye, I can read psychosocial needs? (Select all that Eats with a spoon.
the road signs when we travel.” apply.)
48. A nurse is giving instructions related to client cope with the recent death of the 6-month-old with birth weight of 7 lb 3 Screaming and drawing of the knees up
antibiotic eye drops to the parent of a 5- other parent. When developing a response oz (3.3 kg) who now weighs 12 lb (5.4 to the chest.
year-old with bacterial conjunctivitis. to the parent, the nurse considers that a kg). Stool mixed with blood and mucus.
Which instruction is most important? school-aged child is most likely to do
Wash hands before and after eye drop what? 59. The school nurse is caring for 4 clients 65. A nurse is speaking with the parent of a
instillation. Understand that death is permanent with type 1 diabetes mellitus. Which of toddler who believes the child has a
but be curious about it. the clients should the nurse assess first? hearing deficit. Which findings support
49. Several children seen at a local pediatric 9-year-old who is sweating after recess this suspected diagnosis? (Select all that
clinic are found to have a hemoglobin 54. The emergency department triage nurse is and irritably states, “I’m so hungry!” apply.)
level of 10-11 g/dL (100-110 g/L). Which assessing 4 pediatric clients. Which client Behavior appears withdrawn.
dietary modification would most likely is a priority for further diagnostic workup 60. The nurse is teaching a class on nutrition Monotone speech.
help increase hemoglobin levels in these and definitive care? and feeding practices for young children. Speaks with a loud voice.
clients? 1-year-old with ventriculoperitoneal What should the nurse recommend as the
Ensuring adequate intake of meat, fish, shunt who has “lethargy” and pulse of best snack for a toddler? 66. A nurse is caring for a 2-year-old with a
and poultry. 78/min. Slices of cheese. new diagnosis of strabismus. Which
intervention should the nurse anticipate?
50. The home health nurse is visiting an 55. A nurse is caring for a 1-month-old client 61. In the emergency department, a pediatric Patching the stronger eye.
infant who recently had surgery to repair who is being evaluated for congenital client is placed on mechanical ventilation
tetralogy of Fallot. Which of the hypothyroidism. The nurse should by means of an endotracheal tube. Several 67. A nurse receives report on a group of
following signs of heart failure should the recognize which of the following as hours later, the nurse enters the room and clients. Which client should the nurse
nurse teach the parents to report to the clinical manifestations of hypothyroidism finds the client in respiratory distress. It is assess first?
health care provider? (Select all that in infants? (Select all that apply.) most important for the nurse to take A toddler playing with small toys who
apply.) Difficult to awaken. which of these actions? appears in distress, has circumoral
Cool extremities. Dry skin. Auscultate the client’s lung sounds. cyanosis, and cannot speak.
Puffiness around the eyes. Hoarse cry.
Reduction in number of wet diapers. 62. A child is scheduled to have an 68. The nurse is reviewing teaching with the
Weight gain. 56. A newborn has a large electroencephalogram (EEG). Which parents of a child who has tinea capitis
myelomeningocele. What nursing statement by the parent indicates (ringworm of the scalp) and is newly
51. The parents of a 5-year-old ask the school intervention is priority? understanding of the teaching? prescribed griseofulvin oral suspension
nurse for advice on how to tell their child Cover the area with a sterile, moist “I will wash my child’s hair using and 1% selenium sulfide shampoo. Which
about being adopted. Which dressing. shampoo the morning of the statement by the child’s parent requires
developmentally appropriate thought procedure.” the nurse to intervene?
about adoption by the child does the nurse 57. A nurse is teaching the parent of an 8- “I will discontinue the griseofulvin once
counsel the parents to anticipate? month-old infant who had a febrile 63. A nurse is talking with the parent of a 6- the ringworm stops itching and the
Feels responsible for being placed for seizure about management of future year-old regarding sleep and rest. Which scales go away.”
adoption. fevers. Which instruction is appropriate to information should be included?
include in the teaching? Bedtime hours should be established.
52. The clinic nurse cares for a 4-year-old “Give acetaminophen or ibuprofen
who has been diagnosed with a pinworm every 6 hours to control the fever.” 64. The nurse is assessing an infant with
infection. Which client symptom supports intussusception. Which of the following
this diagnosis? 58. The nurse assesses 4 infants. Which clinical findings should the nurse expect?
Anal itching that is worse at night. assessment finding would require follow- (Select all that apply.)
up by the health care provider? Palpable sausage-shaped abdominal
53. The parent of an 8-year-old client asks the mass.
nurse for guidance on how to help the

69. The parent of a 15-month-old calls the nurse and says that the child developed a rash and mild fever after receiving a routine measles, mumps, rubella, and
varicella (MMRV) vaccine in the pediatric clinic 5 days ago. What is the best response by the nurse?
“What is your child’s temperature right now?”

70. The nurse is teaching a group of new parents about oral hygiene for their children. One of the parents asks, “when
should I take my child to the dentist?”
What would be the best response from the nurse?
“The initial dentist visit should be soon after the child’s first tooth appears.”
71. A 14-year-old is scheduled for surgery to treat scoliosis. The child will be hospitalized for about a week and then discharged home to recuperate for 3-4 weeks before returning to school.
What is the best activity the nurse can recommend to promote age-specific growth and development during this time? Visits from friends.
72. The nurse provides teaching for the parents of a 6-year-old client diagnosed with nocturnal enuresis. Which of the following instructions will the nurse include? (Select all that apply.)
Encourage the child to help change soiled pajamas and linens.
Prepare a calendar with the child for logging wet and dry nights.
Wake the child at a specified time each night to void.

73. The nurse is evaluating a parent’s understanding of home care management for a 2-week-old client after initial cast placement for treatment of congenital clubfoot. Which of the following
statements by the parent indicate a correct understanding? (Select all that apply.)
“I will check my baby’s toes several times a day to ensure that they are pink and warm.”
“My baby will need to have a new
cast applied weekly for 5-8 weeks.”
“When I bathe or diaper my baby, I will be sure to keep the cast dry.”

74. A child in the emergency department had a cast placed on the right arm for a nondisplaced fracture. The client is being discharged home with pain medications. Which statement by the
parent indicates that additional teaching is required?
“A tingling or burning sensation within the first 24-48 hours is not a concern.”

The child received the varicella immunization. The day after the injection, the parent calls the nurses say that the child has discomfort, slight redness, and 2 vesicles at the injection site.
What instruction would be appropriate for the nurse to provide to this parent? Cover the vesicles with a small bandage until they are dry ( 41% )

The nurse is providing education to the parents of a child diagnosed with chronic allergic rhinitis that is triggered
by dust and pollen. Which of the following statements by the parents indicate that the teaching has been effective? Select all that apply.
“We are planning to purchase a vacuum with a high efficiency particulate air filter. ”
“We should place hypoallergenic covers on our child's mattress and pillow.”
“We will dust our furniture with a damp washcloth every week.”

The clinic nurse supervises a student nurse who is preparing to administer routine vaccinations to a child diagnosed with hemophilia period which instructions should the clinic nurse provide to the
student?
Administer vaccines via the subcutaneous route.
Hold firm pressure on the site for 5 minutes.

The nurse just administered routine immunizations to what healthy 15-month-old. What information should the nurse provide the caregivers before they leave the clinic?
Redness at the injection sites and a mild fever are common ( 76% )

A parent calls the nurse Telehealth triage line with concerns about an allergic reaction to something a child ate. Which symptoms should the nurse instruct the patient to assess for to determine if the
child is having an anaphylactic reaction? Select all that apply.
Dyspnea, Lightheadedness, Skin rash (hives), Wheezing

A 12-month-old with Kawasaki disease received IV immunoglobulin (IVIG ) 2 months ago. The child is in the clinic for follow-up and scheduled immunizations. Which vaccine should be delayed?
Select all that apply. Measles, mumps, rubella (MMR) Varicella

A school-age child is brought to the emergency department due to nausea, vomiting, and severe right lower quadrant pain. The child's white blood cell count is 17,000/mm3 ( 17.0 x 109/L). Which
statement by the child is of most concern to the nurse?
“My belly doesn't hurt anymore.” ( 77% )
A nurse is reviewing the laboratory values for a physiotherapy instructions by contacting the HCP about which Answer: Oxygen delivery system, padding
3-year-old client with nephrotic syndrome. The symptom? Answer: The child vomits after on the bed side rails, Suction equipment.
nurse interprets the results to most clearly The nurse assesses a child who has been treated awakening from a nap and 1 hour later.
reflect which physiologic process related to for an acute asthma exacerbation. Which client A nurse is teaching the parent of a child who
nephrotic syndrome? Answer: Glomerular assessment is the best indicator that treatment A nurse is caring for a 3-month-old infant who has a new diagnosis of absence seizures. Which
injury has been effective? Answer: oxygen as bacterial meningitis. Which clinical findings statement by the parent indicates understanding
of the teaching? Answer: “My child May
saturation has increased from 88% to 93%. support this diagnosis? Select all that apply.
stare and seem inattentive.”
The nurse has provided teaching about home Answer: Frequent seizures, High-pitched
care to the parent of a 10-year-old with cystic When performing developmental screenings in cry, Poor feeding, Vomiting. The nurse cares for a child newly diagnosed
fibrosis. Which of the following statements by the well-child clinic, the registered nurse with cystic fibrosis. What should be included in
the parent indicate that teaching has been understands that which child is at highest risk A 4-year-old boy is diagnosed with Duchenne the clients multidisciplinary plan of care to be
effective? Select all that apply. Answer: “i will of developing autism spectrum disorder? muscular dystrophy. Which nursing teaching is discussed with the parents? Select all that
give my child pancreatic enzymes with all Answer: 4-year-old whose 10-year-old sibling most appropriate to reinforce for the child’s apply. Answer: Aerobic exercise, Chest
meals and snacks”, “I will increase my has the disorder. parents? Answer: Remove throw rugs from physiotherapy, Financial needs.
child’s salt intake during hot weather”, “Our the home.
child will need a high-carbohydrate, high- The nurse has assessed 4 children. Which A 2-month-old recently diagnosed with
protein diet.” finding requires immediate follow-up with the Which pediatric respiratory presentation in the developmental dysplasia of the hip (DDH) is
health care provider? Answer: child who had a emergency department is a priority for nursing beginning treatment with a Pavlik harness.
care? Answer: Client with an acute asthma Which instructions should the nurse provide to
Several children are brought to the emergency surgical repair of hypospadias earlier today
exacerbation but no wheezing. the parents? Select all that apply. Answer:
room after a boating accident in which they with no urinary output in the past two hours.
“Dress the child in a shirt and knee socks
were thrown into the water. The children are A 7-month-old infant is admitted to the unit under the straps.”, “Lightly massage the
now 6 hours post admission to the clinical A child is brought to the school nurse after with suspected bacterial meningitis after skin under the straps daily.”, “Place the
observation unit.Which client should the nurse having a permanent tooth knocked out during receiving an initial dose of antibiotics in the diaper under the straps.”
evaluate first? Answer: client who received gym class. Which action by the nurse is emergency department. Frequent assessment of
CPR for 2 minutes on the scene and whose appropriate?Answer: gently rinse the tooth which of the following is most important in A home health nurse is managing care for an
respiratory rate has now dropped from with sterile saline and reinsert it into the the plan of care? Answer: Fontanel adolescent client with cystic fibrosis. Which of
61/min to 18/min gingival cavity. assessment the following potential complications should
the nurse consider when developing a nursing
A nurse is caring for a child who is receiving The nurse recieves change of shift report on 4 The nurse is assessing a 3-year-old client in the care plan? Select all that apply. Answer:
oxygen at 2 L/min by nasal cannula and clients. Which client should the nurse assess emergency department and finds dyspnea, high Chronic hypoxemia, Frequent respiratory
observes the current oxygen saturation and first? Answer: 2-year-old with suspected fever, irritability, and open-mouthed drooling infections, vitamin deficiencies.
with leaning forward. The parents report that
pulse plethysmographic waveform on the pulse epiglottis
the symptoms started rather abruptly. The A 2-year-old child is brought to the emergency
oximeter. Which intervention should be the client has not received age-appropriate department for a severe sore throat and fever of
nurse’s initial action? Answer: verify the The nurse is admitting a 4-year-old diagnosed vaccinations. Which set of actions should the 102.9 F (39.4 C). The nurse notes that the child
position and integrity of the finger probe with Wilms tumor. The child is scheduled for a nurse anticipate? Answer: Intubation in the is drooling with distressed respirations and
right nephrectomy in the morning. Answer: operating room with a prepared inspiratory stridor. What action should the
The summer camp nurse and parent of a 9-year- Instructions not to palpate the abdomen tracheotomy kit standing by. nurse take first? Answer: Position the child
old with juvenile idiopathic arthritis (JIA) are in tripod position on the parents lap.
discussing appropriate physical activities for the A nurse is caring for a child with acute The nurse assessed a a pediatric client who was
child. Which of the following activities should glomerulonephritis. Frequent monitoring of diagnosed with diarrhea caused by Escherichia The triage nurse is assessing an unvaccinated
be included? Select all that apply Answer: which of the following is a priority? Answer: Coli. The nurse is most concerned with which 4-month-old infant for fever, irritability, and
stationary bicycling, swimming, yoga. blood pressure finding? Answer: Petechiae noted on the open-mothers drooling. After the infant is
trunk successfully treated for epiglottis, the parents
A 6-month-old client has been diagnosed with The nurse taught the caregiver of a child with wonder how this could have been avoided.
The nurse is preparing for the admission of a 9- Which response by the nurse would be the
cystic fribrosis. Which of the following would ventriculoperitoneal (VP) shunt about when to
year-old client with new-onset tonic-clonic most appropriate? Answer: “Most cases of
be appropriate for the registered nurse to teach contact the health care provider (HCP). The seizures. It is important for the nurse to ensure epiglottitis are preventable by standard
the parents? Answer: perform manual chest caregiver shows understanding of the that what is in the room? Select all that apply. immunizations.”
A child with autism spectrum disorder is being admitted to an acute care unit. Which is the most important nursing action? Answer: Placing a child in a private room away from the nurses
station

The school nurse assesses and 8-year-old with a history of asthma. The nurse notes mild wheezing and coughing. Which action should the nurse perform first? Answer: Asses the clients peak
expiratory flow.

The nurse is assessing a 4-week-old infant during a routine office visit. Which assessment finding is most likely to alert the nurse to the presence of right hip developmental dysplasia? Answer:
Presence of extra gluteal folds on the right side

The parent of a newborn is concerned about the possibility of the child developing hip dysplasia. Which intervention should the nurse encourage to help reduce the risk in this newborn? Answer:
Swaddle the infant with hips flexed and abducted.

A 3-month-old with developmental dysplasia of the hip (DDH) is being fitted for a Pavlik harness. Which statement made by the parent indicates a need for further instruction? Answer: I will
adjust the harness straps every 3-5 days.

The nurse has been providing care for the past month to a 7-year-old client recently diagnosed with type one diabetes mellitus. Initially, the family seemed devastated about the diagnosis and the
client’s parents stated, “our lives will never be the same.” which statement made by the parent indicates that nursing interventions and education have been effective?
“we are working to manage this disease so that it cannot control our child's life .” ( 74% )

The nurse is planning a client care conference with the parents of a 3-year-old with newly diagnosed type 1 diabetes mellitus. What is the priority outcome for the caregivers?
Knowing how to keep blood sugar stable

The school nurse evaluates a 9-year-old who is sweating, trembling, and pale. The client has type 1 diabetes managed with insulin glargine and NPH. What is the most appropriate action by the nurse?
Provide 4 oz (120 mL) of a regular soft drink

A 6-year-old client was diagnosed with type 1 diabetes mellitus 2 years ago. The nurse would like to encourage the client to participate in disease management. Which of the following diabetes care
tasks are important for their child to perform? Select all that apply.
Choose insulin injection site with parental oversight of rotation schedule
Push plunger of insulin syringe after apparent inserts and stabilizes the needle
Select and clean the site for finger-stick blood glucose testing
Verbalize two or three signs and symptoms of hypoglycemia

The nurse is performing of physical examination on a 10-year-old client with abdominal discomfort. Which actions would be appropriate during the examination? Select all that apply.
Ask the client to describe the chief symptom
Conduct a head to toe assessment in the same manner as in adult assessment
Honor the clients request to be examined without parent present

The registered nurse is teaching the parent of a 6-year-old about behavioral start strategies for treating fecal incontinence due to functional Constipation. Which statement by the parent indicates a
need for further teaching?
“I will give my child a reward for each bowel movement while sitting on the toilet.”
A newborn is being evaluated for possible esophageal atresia with tracheoesophageal fistula. Which finding is the nurse most likely to observe?
Choking and cyanosis during feeding

A parent of a 7-month-old reports that the child has been crying and vomiting with a distended belly for the past 4 hours. The infant is now lying quietly in the parent's arms with a pulse of 200/min
and respirations of 60/min. Which of the following components of SBAR (situation, background, assessment, recommendation /read-back) communication is most important for the nurse to report to
the health care provider?
Client is now lethargic with abnormal vital signs

An overweight toddler is diagnosed with iron deficiency anemia. Which is the most likely explanation for the anemia?
Excessive intake of milk

The school nurse creates a cafeteria menu for a newly enrolled child with celiac disease. Which lunches would be appropriate for this child? Select all that apply.
Grilled chicken, baked potato, and strawberry yogurt, Mexican corn tacos with ground beef and cheese, Rice noodles with chicken and broccoli

The client nurse is caring for several clients during well-child visits. The nurse should recognize each client as most as being the most at risk for anemia?
3-month-old infant born at preterm gestation who is exclusively bottle-fed with breast milk

A nurse is evaluating a client's understanding about interest formula preparation period which of
the following clients statements indicate proper understanding? Select all that apply.
“I must wash the top of concentrated formula cans before opening.”
“On use prepared formula should be kept in the refrigerator and discarded after 48 hours.”

The nurse plans care for a pediatric client who has just undergone a cleft palate repair. which of the following interventions should the nurse include in the plan of care? Select all that apply.
Assist and encourage caregivers to hold and comfort the child
Position the child supine with an elevated head of bed after feedings
Remove elbow restraints per policy for skin and circulatory assessment

The nurse is reviewing anticipatory guidance with the parents of a 6-month-old infant with phenylketonuria. Which statements by the nurse are appropriate? Select all that apply.
“a low phenylalanine is required.”
“meat and dairy products should not be introduced into the diet.”
“special infant formula is required.”

The nurse is assessing a 3-month-old infant who was admitted to the floor 18 hours ago after undergoing surgical repair of a cleft lip. Which assessment finding would cause the nurse to be
concerned?
The client is prone while playing with the parent
The parent of a newborn is concerned about the child’s favorite stuffed 9. A nurse on a pediatric unit is
the possibility of the child developing hip The nurse provides discharge teaching for the animal, Establish a daily schedule reviewing interventions for a
dysplasia. Which intervention should the parents of a child newly diagnosed with to similar to the child’s home toddler with a practical nurse who
nurse encourage to help reduce the risk in this hemophilia A. Which statements by the routine, Provide frequent will be caring for this child. Which
newborn? parents indicate that teaching has been opportunities for play and of the following are appropriate
Swaddle the infant with hips flex an effective? Select all that apply. activity activities to minimize the effect of
abducted “our child should wear a medical alert hospitalization on a toddler? Select
bracelet at all times.” 4. The parents of a hospitalized 3- all that apply. Integrate preferred
The nurses triage in a 7-year-old with sickle “we should avoid giving our child over-the- month-old have to leave the infant snack foods in the day’s routine,
cell crisis. The client is short of breath and counter medicine containing aspirin.” while they work. One parent fears Plan a quiet play prior to usual
vomiting and has severe generalized body and “we should encourage a non-contact sport that the baby will cry as soon as nap time, Provide 1 or 2 options
joint pains. Which assessment finding such as swimming.” they walk out. The nurse teaches when choosing toys
requires the most immediate intervention? both parents about separation
Enlarged spleen on palpitation What is the best activity for school-aged child anxiety. Which statement by the 10. A nurse is discussing the concept
hospitalized for vaso-ooclusive sickle cell parent indicates that the teaching of parallel play with parents of
The nurse is completing chemotherapy crisis? has been effective? At this age, my toddlers. Which statement should
education with an adolescent client. The client Watching a favorite movie baby will not cry because we are the nurse include to describe this
states “I would rather continue to have cancer leaving type of play? Children play near
than lose all of my hair.” how should the A nurse manager on a pediatric oncology unit other children but without
nurse respond? is leading a discussion about personal coping 5. A nurse in a pediatric clinic is significant interaction
“how do you feel about meeting other teens strategies that nurses can use to remain performing a physical examination
who are receiving chemotherapy?” effective when caring for dying children. of a 30-month-old child. Which 11. The nurse is caring for a
What should be included in this discussion? finding requires further evaluation? hospitalized 6-month-old client.
The nurse is planning teaching for the parents Attending a child’s memorial service can Current weight is 6 times greater Which of the following
of a child newly diagnosed with hemophilia be helpful than birth weight interventions should the nurse
will include information about which long- implement to provide
term complication? The parents of a 5-year-old ask the school 6. The nurse is providing health developmentally appropriate care
Joint destruction nurse for advice on how to tell their child promotion education to the parent for this client? Select all that apply.
about being adopted. Which developmentally of a toddler. Which statement by the Adhere to the child’s home
A teenage client with sickle cell disease is appropriate though about adoption by the parent requires the nurse to clarify routine when possible during
admitted with a diagnosis of crisis. The child does the nurse counsel the parents to teaching? If my child refuses a hospitalization, Encourage
client's current prescription is morphine 2 mg anticipate? Feels responsible for being meal, I will have them stay at the parents to bring the child’s
intravenous push every 4 hours PRN. The placed for adoption table until they eat half the food favorite toy from home, Promote
client appears comfortable while watching a quiet sleep environment with
television and tells the nurse “I have severe 1. What play behavior would the nurse 7. What communication strategies reduced stimuli, Provide a
intolerable pain,” and rates it a “10.” what be most likely to observe in a group would the nurse have in place when parent’s shirt for the child to hold
action should the nurse take? of a 4-year-old children? Children establishing rapport with the during procedures
Called the HCP for patient control playing and borrowing blocks caregiver and an 8-year-old during a
analgesia (PCA) at a higher dose of the from each other without directing health history interview? Select all 12. A nurse is discussing parallel play
same drug others that apply. Allow the child to with the parent of a 2-year-old.
describe their issue, Maintain an Which statement by the parent
A 6-month-old infant is brought to the 2. The nurse is conducting a eye level position when speaking indicates understanding of the
emergency department after experiencing psychosocial developmental with the child, Use language that discussion? I set out a basket of
vomiting and diarrhea for 4 days. Which checkup on a 2-year-old child. both the child and caregiver can various balls in the backyard
prescription from the health care provider is What is the priority assessment understand when other children come to play
priority ? finding that should be reported to
IV normal Saline bolus 20 mL/kg over 1 the primary health care provider? 8. The school nurse is conducting an 13. The nurse in a clinic is obtaining a
hour Does not talk or respond to being educational session for middle developmental history of an 18-
talked to or read to school teachers that is designed to month-old during a well-child visit.
The nurse is caring for pediatric client with heighten awareness of school Which activities should the child be
end stage leukemia who is on comfort care 3. The nurse on a pediatric unit is bullying. The nurse recognizes that able to perform? Select all that
and is unresponsive. The child's parents ask, caring for a preschooler who further instruction is needed apply. Goes up stairs while
“how can you tell if my child is in pain?” exhibits separation anxiety when when one of the teachers makes holding a hand, Turns 2 pages in
which of these would the nurse describe as the parents go to work. Which which comment? Bullying is a a book at a time
signs of discomfort? Select all that apply. interventions should the nurse normal part of childhood growth
Facial grimacing, groaning, knees bent up implement? Select all that apply. and development 14. A nurse is performing an
near chest Encourage the parents to leave assessment of a 12-month-old child.
Which of the following findings 21. The nurse is providing care to a 9- The nurse explains that the client Which statement by the parents
would the nurse expect? Select all year-old client who is awaiting will receive “medicine under the indicates a need for further
that apply. Birth weight is tripled, surgery. Which intervention is skin,” and the client is visibly instruction? When we can’t be
Fully developed pincer grasp, Sits developmentally appropriate for this anxious. Which nursing watching, we put our child in a
from a standing position client’s plan of care? Discuss the intervention is appropriate? Ask the mobile child walker
procedure with the client using child to count to 10 during
15. A nurse is discussing the fine simple diagrams with correct injection 35. The nurse is caring for a 10-year-
motor abilities of a 10-month-old anatomical terminology old diagnosed with osteomyelitis.
infant with the infant’s parent. 29. The clinic nurse is asked by the What is the best activity the nurse
Which are developmentally 22. A nurse is planning to complete a mother of a 15-month-old, “I am can suggest to promote age-specific
appropriate skills for an infant of physical examination of a toddler. worried about my child’s sucking growth and development during
this age? Select all that apply. Which approach is an appropriate and its effects on tooth alignment. hospitalization? Provide missed
Grasps a small doll by the arm, intervention by the nurse? What should I do?” What is the schoolwork
transfers small objects from hand Encourage the parent to be nurse’s best response? The risk for
to hand, Uses a basic pincer grasp involved with the child misaligned teeth occurs when 36. A 5-year-old child is receiving
thumb sucking persists after morphine sulfate for pain. Which
16. The nurse is caring for a preschool- 23. A 15-year-old parent brings a 4- eruption of permanent teeth statement by the caregiver indicates
age child whose grandparent died 3 month-old infant for a well-baby that further teaching is necessary?
days ago. What intervention is checkup. The parent tells the nurse 30. The clinic nurse reviews teaching My child is playing and so does
inappropriate? Avoid mentioning that the baby cries all the time; the provided to the parent of a child not need pain medication
the loved one’s death in the parent has tried everything to keep being considered for growth
child’s presence the infant quiet but nothing works. hormone replacement therapy at 37. The nurse is performing a well-
What is the priority nursing action? home. Which statement by the child assessment on a sleeping 2-
17. The registered nurse has completed Assess the infant’s pattern and parent indicates that teaching has month-old client. Organize the
a well-baby assessment of an 18- frequency of crying been effective? Treatment will assessment in the correct order
month-old. Which assessment require a daily injection under based on the developmental age of
findings prompted the nurse to 24. The parents of a 4-year-old tell the my child’s skin the client. Observe skin color and
make a referral for a formal nurse that the child won’t go to respiratory pattern, Auscultate
developmental screening test? Uses sleep at night due to the fear of 31. A 9-year-old has terminal cancer, heart and lungs, Palpate
2 words, cannot hold a cup, can tigers living under the bed. Which but the parents do not want the child fontanelles and abdomen, Assess
seat self in a small chair response by the nurse is most to know the prognosis. The child pupillary response, Elicit Moro
helpful? Night fears are common has been asking questions such as reflex
18. The nurse is educating a group of at this age. Look under the bed what dying is like and whether the
parents about ways to decrease the with your child child will die. Which action by the 38. The nurse is assessing a 8-month-
risk of sudden infant death nurse is most appropriate? Tell the old client during a well-child visit.
syndrome. Which of the following 25. A 10-year-old weighs 99 lb (44.9 parents about the child’s Which assessment finding should
recommendations should the nurse kg) and has a BMI of 24.8 kg/m2 questions the nurse report to the health care
suggest? Select all that apply. (>95th percentile). Which is the provider? Infant’s head stays
Breastfeed the infant, Ensure the most important assessment for the 32. The nurse is assessing a 3-month- behind the shoulders when raised
infant’s vaccinations are updated, nurse to make before initiating a old during a well-child visit. Which from a supine to a sitting position
Maintain a smoke-free weigh loss plan? Family’s developmental finding should the
environment, Provide a firm readiness for change nurse expect to observe in the 39. The parent of a 1-year-old says to
sleeping surface for the infant client? Infant kicks legs, smiles, the nurse, “I would like to start
26. The nurse assessing a 2-year-old and coos when a familiar face toilet training my child as soon as
19. The nurse is providing teaching to should expect the child to be able to comes into view possible.” What information does
the parents of a child with Marfan perform which actions? Select all the nurse provide to the parent that
syndrome. Which topic is the that apply. Build a tower, Say own 33. The public health nurse has correctly describes a child’s
priority for the nurse to address? name, Walk without help received a referral to make a readiness for toilet training? Your
Avoiding participation in contact follow-up home visit to a 1-year-old child may be ready to start toilet
sports 27. The nurse cares for a 4-year-old recently diagnosed with failure to training when able to
who is on long term, strict bed rest. thrive (FTT). Which intervention is communicate and follow
20. A nurse in a clinic is talking with a Which toy is most appropriate to the priority nursing action for this directions
parent about the onset of puberty in provide diversion and minimize child? Observe the child feeding
boys. What is the first sign of developmental delays? Puppets 40. The nurse is reviewing the plan of
pubertal change that occurs? 34. The nurse is discussing child safety care for a 4-year-old client who will
Testicular enlargement 28. The nurse prepares a 7-year-old with the parents of a 12-month-old receive daily dressing changes for
client for an influenza injection. who is just beginning to walk. an infected leg wound. Which of the
following interventions should the be obtained from other sources? Select all that C. Rapid Streptococcal antigen
nurse include in the plan of care for 45. The parents of a 2-year-old client apply. Calcium, vitamin D test
a preschool-aged child? Select all ask how they can help their child D. Skin Biopsy
that apply. Allow the child’s cope with hospitalization. Which of A newborn had a bowel resection with 2. Which discharge teaching
parents to stay during the the following suggestions should temporary colostomy for Hirschsprung's instructions should the nurse
procedure, Emphasize that the nurse give the parents? Select disease. The nurse should alert the health care provide to the parents of a 2-year-
dressing changes are not all that apply. Follow as many provider (HCP) for which assessment finding old with group A streptococcal
punishment for misbehavior, home routines as possible, Sleep post operatively? Stoma is Gray-tinged at pharyngitis? Select all that apply
Encourage the child to voice in the child’s hospital room at the edges but pink at the center on A. Complete all the antibiotics
questions and concerns about the night, Take child on regular visits postoperative day 5 even if your child is feeling
procedure, Have the child place to the playroom better
bandages on a doll when a nurse in the neonatal intensive care unit B. Cool liquids and soft diet are
reinforcing education. 46. A 10-year-old is implementing discovers a cyanotic newborn with excessive recommended
behavioral strategies to manage frothy mucus in the mouth. What should be C. Keep your child at home from
41. The nurse is reviewing the medical nocturnal enuresis. The client tells the nurse’s first action? Suction the infant’s daycare for at least a week
record of a 4-year-old client with the nurse, “I want to go to sleep mouth D. Replace your child’s
failure to thrive. Which of the away camp during the summer, but toothbrush 24 hours after
following risk factors likely if I have an accident, I’m afraid that The clinic nurse supervises a graduate nurse starting antibiotics
contribute to the client’s condition? other kids with tease me. What is who is teaching the parents of a 2-year-old E. Throat lozenges may soothe
Select all that apply. One parent is the best response by the nurse? We with acute diarrhea about home management. your child’s sore throat
incarcerated for spousal abuse, can ask your health care provider The nurse would need to intervene when the 3. The emergency nurse is triaging
One parent was diagnosed with about a medication trial that may graduate nurse provides which instruction? four clients. Which client should the
anorexia nervosa prior to having help “follow the bananas, rice, Apple sauce, and nurse assess first?
children, Parents are concerned toast diet for the next few days.” A. 6-month-old client who is
about not having enough money The nurse assesses a child with constipated, cannot hold the
to buy food intussusception. Which assessment findings the nurse is reinforcing education with the head up, and has a weak cry
require priority intervention? Abdominal parents of a 2-year-old child about diet after eating honey
42. The parent of a 2-year-old tells the rigidity with guarding choices to promote growth. the family B. A year-old client who has
nurse at the well-child clinic, “I am observes a strict vegan diet. Which of the elbow pain, swelling, and
concerned because my child does During a routine assessment of a following statements by the nurse are deformity after falling from a
not like to be cuddled, does not developmentally normal 18-month-old the appropriate? Select all that apply. playground swing
respond when called by name, and parent expresses concern about the small “it is important to feed your child fortified C. A 6-year-old client who has a
does not make eye contact when amount of food the child consumes. What is breads and cereals to help with iron fever and reports right-sided
being fed.” What is the priority the nurse’s priority intervention? intake.” abdominal pain and difficulty
question for the nurse to ask when Educate the parent about physiologic “try to pair foods high in iron with foods tolerating food
completing the health history? How anorexia high in vitamin C to aid iron absorption.” D. A 9-year-old client who is
many words can your child say? “your child may require calcium and lethargic and reports having
A parent brings a 6-month-old child to the vitamin D supplementations due to lack of painful urination and back pain
43. A distraught parent informs the Primary Health care provider after the child dairy intake.” 4. The clinic nurse provides teaching
nurse of a bleeding in a 1-year-old abruptly started crying and grabbing for a parent of a child diagnosed
girl. What is an appropriate intermittently at the abdomen. The client stool the clinic nurse is interviewing the parents of with scabies. Which instructions
response by the nurse after has a red, currant Jelly appearance. What a 6-month-old client about the infants diet. should the nurse include in the
assessing a small amount of bloody intervention does the nurse anticipate? Give Which statement by the parents is most teaching plan? Select all that apply
mucus in the newborn’s diaper? We air (pneumatic) enema concerning? “infant oatmeal sweetened with A. All the persons in close
will continue to monitor the fresh honey is our baby's favorite contact with the child need
amount, color, and consistency of The nurse is educating the parents of a 6- breakfast.” treatment
the drainage month-old about introducing solid foods into B. Apply the permethrin cream
the infant's diet. Which parental statement 1. The clinic nurse assesses an 8-year- to all skin surfaces
44. A 15-year-old client with type 1 indicates a need for further teaching? “I can old client who reports a sore throat C. Discard the child’s stuffed
diabetes mellitus (DM) is admitted offer of variety of foods within the first and has a bright red, pruritic rash on animals
to the pediatric intensive care unit week of introducing solids.” the chest that feels like bumps and D. Fumigate all the living areas in
with a blood glucose level of 460 looks like sunburn. Which the home
mg/dL (25.5 mmol/L). The nurse The nurse is reinforcing discharge teaching diagnostic tool does the nurse E. Wash the child’s clothing
understands that which factor is for the parents of a 1-year-old with a newly anticipate the health care provider and bedding in hot water
contributing to this client’s diagnosed cow’s milk allergy. Which will prescribe? 5. A nurse manager on a pediatric
noncompliant behavior? Client’s nutrients normally provided by milk should A. Allergy skin test oncology unit is leading a
psychosocial developmental stage B. Complete blood count discussion about personal coping
strategies that nurses can use to C. “I will spray the house with 12. The nurse just administered routine E. Request an order for cough
maintain effective when caring for insecticide to control the immunization to a healthy 15- suppressant
dying children. What should be problem” month-old. What information 16. An adolescent client with a sore
included in the discussion? D. “I will throw away stuffed should the nurse provide the throat is diagnosed with infectious
A. Attending a child’s memorial animals and toys that cannot be caregivers before they leave the mononucleosis. Which comment by
service can be helpful washed” clinic? the caregiver would alert the nurse
B. Avoid expressing personal 9. A nurse is caring for a 6-year-old A. Call the office of the toddler’s that additional instruction is
feelings of grief or loss directly client with tonsilitis. Which further temperature is higher than 100 necessary?
with the family assessment finding requires F A. “I need to go to the
C. Personal contact with the immediate intervention? B. Fussiness and anorexia are pharmacy to pick up an
family of the deceased should A. Dry mucous membranes common for 1 week after antibiotic”
end after they leave the B. Prescence of trismus immunizations B. “It is acceptable for my child
hospital C. Pulling at the ears C. Redness at the injection sites to have ibuprofen for
D. The length of daily exercises D. Sandpaper-like skin rash and a mild fever are common discomfort or fever”
should be increased 10. The nurse provides discharge D. The toddler’s activity level C. “My child will be on bed rest
6. A parent calls the nurse telehealth teaching for the parents of a child should be restricted for 24 with a few activities for the
triage line with concerns about an newly diagnosed with hemophilia hours next 2 weeks”
allergic reaction to something a A. Which statements by the parents 13. A school age child is brought to the D. “Participation in soccer
child ate. What symptoms should indicate that teaching has been emergency department due to practice will not be allowed for
the nurse instruct the parent to effective? Select all that apply nausea, vomiting, and severe right the next month”
assess to determine if the child is A. “A high-calorie, high-protein lower quadrant pain. The child’s 17. A nurse is caring for a school-age
having an anaphylactic reaction? diet is best for the child” white blood cell count is client with fever, somnolence, and a
Select all that apply B. “It is extremely important that 17,000/mm3 (17.0x10 9/L). Which skin rash from suspected
A. Dyspnea we do not allow our child to statement by the child is of most meningococcal meningitis. Which
B. Fever become dehydrated” concern to the nurse? of the following interventions
C. Lightheadedness C. “Our child should wear a A. “I am hungry, and they will should the nurse plan to implement
D. Skin rash (hives) medical alert bracelet at all not let me eat” for this client? Select all that apply
E. Wheezing times” B. “I do not like hospitals and A. Elevate the head of the bed
7. A nurse is teaching the parent of a D. “We should avoid giving our want to go home” 30 degrees
6-year-old with a urinary tract child over-the counter C. “I’m so tired” B. Implement seizure
infection (UTI) how to avoid repeat medicine containing aspirin” D. “My bell doesn’t hurt precautions
infections. Which statements by the E. “We should encourage a anymore” C. Keep a mask on the client at all
patient indicate that the teaching has noncontact sport such as 14. The nurse is triaging a 7-year-old times
been effective? Select all that swimming” with sickle cell crisis. The client is D. Minimize environmental
apply 11. A teenage client with sickle cell short of breath and vomiting and stimuli
A. “I just bought my child new disease is admitted with a diagnosis has severe generalized body and E. Place client in a room with
nylon panties” of crisis. The client’s current joint pain. Which assessment negative pressure air flow
B. “I will make sure my child prescription is morphine 2 mg finding requires the most 18. A 2-year-old is diagnosed with
does not hold urine” intravenous push every 4 hours prn. immediate intervention? atopic dermatitis (eczema). Which
C. “I will not give my child any The client appears comfortable A. Blood work showing anemia instructions should the nurse teach
more bubble baths” while watching television and tells B. Enlarged spleen on palpation the parents? Select all that apply
D. “I will teach my child to wipe the nurse “I have severe intolerable C. Right arm weakness A. Apply emollient immediately
from the front to the back” pain”, and rates it at a “10”. What D. Swelling of hands and feet after a bath
E. “I will use antibacterial soap action should the nurse take? 15. The nurse is planning care for a 3- B. Dress child in wool pajamas
for bathing my child” A. Call the clients healthcare year-old who is admitted with C. Give tepid baths with mild
8. A school nurse is educating the provider (HCP) to obtain suspected pertussis infection. soap
parent of a young client with ibuprofen prescription for the Which instructions will the nurse D. Keep child’s nails well-
pediculosis captitis. Which pain include in the plan of care? Select trimmed
statement by the parent indicates B. Call the HCP for the patient- all that apply E. Thoroughly rub the skin after
understanding of the teaching? controlled analgesia (PCA) at A. Institute droplet precautions baths
A. “I will launder recently worn a high dose of the same drug B. Monitor for signs of airway 19. A 2-year-old child is brought to the
clothing, sheets, and towels C. Contact the HCP who issued obstruction emergency department for a severe
in hot water” the prescription to switch to C. Offer small amounts of fluids sore throat and fever of 102.9 F
B. “I will make sure all eating meperidine frequently (39.4 C). The nurse notes that the
utensils are placed in the D. Realize the client is exhibiting D. Place the child in a negative child is drooling with distressed
dishwasher” signs of addictive behavior and pressure isolation room respirations and inspiratory stridor.
needs an appropriate consult
What action should the nurse take D. “There is nothing you could Duchenne muscular dystrophy? 30. When monitoring an infant with
first? have done, the important thing Select all that apply left-to-right sided heart shunt,
A. Assess an accurate temperature is that your child is safe now” A. Frequently trips and falls at which findings would the nurse
with a rectal thermometer 23. The nurse has provided teaching home expect during the physical
B. Directly examine the throat for about home care to a parent of a 10- B. Has painful knees and elbows assessment? Select all that apply
the presence of exudates year-old with cystic fibrosis. Which in the morning A. Clubbing of fingertips
C. Obtain intravenous access for of the following statements by the C. Places hands on the thighs to B. Cyanosis when crying
anticipated steroid parent indicates that teaching has push up to stand C. Diaphoresis during feedings
administration been effective? Select all that D. Suddenly rigidly extends the D. Heart murmur
D. Position the child in tripod apply arms and legs E. Poor weight gain
position on the parents’ lap A. “Chest physiotherapy is E. Walks on tiptoes and has 31. A nurse is teaching the parent of a
20. A 4-year-old diagnosed with administered only if disproportionately large child who has a new diagnosis of
Duchenne muscular dystrophy. respiratory symptoms worsen” calves absence seizures. Which statement
Which nursing teaching is most B. “I will give my child 27. A 3-month-old child with by the parent indicates
appropriate to reinforce for this pancreatic enzymes with all developmental dysplasia of the hip understanding of the teaching?
child’s parent? meals and snacks” (DDH) is being fitted for a Pavlik A. “My child may experience
A. Increase intake of foods high C. “I will increase my child’s harness. Which statement made by incontinence”
in iron salt intake during the hot the parent indicates a need for B. “My child may seem
B. Lift weights to strengthen weather” further instruction? confused afterwards”
weak muscles D. “Our child will need a high- A. “I should leave the harness on C. “My child may stare and seem
C. Remove the rugs from the carbohydrate, high-protein during diaper changes” inattentive”
home diet” B. “I will adjust the harness D. “My child will notice unusual
D. Take the muscle relaxant E. “We will limit our child’s straps every 3-5 days” odors prior to the event”
Baclofen on time participation in sports C. “I will inspect the skin under 32. The summer camp nurse and parent
21. A home health care nurse is activities” the straps 2-3 times daily” of a 9-year-old with juvenile
managing care for an adolescent 24. The nurse is assessing a 4-week-old D. “The harness should keep my idiopathic arthritis (JIA) are
client with cystic fibrosis. Which of infant during a routine office visit. baby’s legs bent and spread discussing appropriate physical
the following potential Which assessment finding is most apart” activities for the child. Which of the
complications should the nurse likely to alert the nurse for the 28. A child with autism spectrum following activities should be
consider when developing a nursing presence of right hip development disorder is being admitted to an included? Select all that apply
care plan? Select all that apply dysplasia? acute care unit. Which is the most A. Dodgeball
A. Chronic hypoxemia A. Decreased right hip adduction important nursing action? B. Reading a book
B. Diabetes insipidus B. Presence of extra gluteal A. Placing the child in a private C. Stationary bicycling
C. Frequent respiratory folds on right side room away from the nurse’s D. Swimming
infections C. Right leg longer than the left station E. Yoga
D. Obesity leg B. Placing the child in a private 33. A 7-month-old infant is admitted to
E. Vitamin deficiencies D. Right pelvic tilt with lordosis room near the playroom the unit with suspected bacterial
22. The triage nurse is assessing an 25. The school nurse evaluates a 9- C. Placing the child in a semi- meningitis after receiving an initial
unvaccinated 4-month-old infant for year-old who is sweating, private room near the nurse’s does of antibiotics in the emergency
fever, irritability, and open-mouthed trembling, and pale. The client has station department. Frequent assessment of
drooling. After the infant is type 1 diabetes managed with D. Placing the child in a semi- which of the following is most
successfully treated to epiglottis, the insulin glargine and NPH. What is private room with another important in the plan of care?
parents wonder how this could have the most appropriate action by the child with autism spectrum A. Babinski reflexes
been avoided. Which response by nurse? disorder B. Fontanel assessment
the nurse would be most A. Administer scheduled dose of 29. The nurse is preparing for the C. Pulse pressure
appropriate? NPH insulin admission of a 9-year-old client D. Pupillary light response
A. “it’s most important to know B. Give emergency glucagon IM with a new-onset tonic-clonic 34. The nurse taught a caregiver of a
for sure what could have injection seizures. It is important for the child with a ventriculoperitoneal
caused this episode” C. Give peanut butter and nurse to ensure that what is in the (VP) shunt about when to contact
B. “Most cases of epiglottis are crackers room? Select all that apply the health care provider (HCP). The
preventable by standard D. Provide a 4 oz (120 mL) of a A. Oral bite prevention device caregiver shows understanding of
immunizations” regular soft drink B. Oxygen delivery system the instructions by contacting the
C. “We are still waiting for the 26. The nurse is assessing a 4-year-old C. Padding on the bed HCP about which symptom?
formal report from the boy in a pediatric clinic. Which siderails A. A temperature of 99 F (37 C)
microbiology laboratory” behaviors by the client would D. Soft arm and leg restraints that occurs during the evening
concern the nurse for possible E. Suction equipment
B. The child cannot recall items C. Lateral curvature to the client also has old fractures in
eaten for lunch during previous spine noted on examination multiple stages of healing but no The most recent laboratory results for a 12-
day of a 10-year-old girl bruising, abrasions, or redness of month-old who is HIV-positive show a CD4
C. The child vomits after D. Presence of an S3 heart sound the skin. Which nursing lymphocyte count of 500/mm3 and a CD4
awakening from a nap and 1 in a 2 -year-old intervention should be included in lymphocyte percentage of 10%. The nurse
hour later 38. A 3-month-old who weighs 8.8 Ib. the plan of care? anticipates administering which
D. The VP shut is palpated along (4 kg) has just returned to the A. During diaper changes, immunizations? Select all that apply.
the posterior lateral portion of intensive care unit after surgical carefully lift the infant by the Haemophilus Influenzae type B (Hib),
the skull repair of a congenital heart defect. ankles hepatitis A (Hep A), pneumococcal
35. The nurse is planning care for a Which finding by the nurse should B. Lift from under the arms when conjugate vaccine (PCV)
child being admitted for Kawasaki be reported immediately to the picking up the infant
disease and should give priority to health care provider? C. Obtain blood pressure The nurse is performing a physical
which nursing intervention? A. Chest tube output of 50 mL manually to avoid cuff over- assessment on a 2-year-old with cold
A. Apply cool compress to the in the past hour tightening symptoms and a fever at home of 101.7 F
skin of the hands and feet B. Heart rate of 150/min D. Request a social work (38.7 C). The parent is concerned about the
B. Monitor for a gallop heart C. Temperature of 97.5 F (36.4C) consultation to assess for child child's ability to cooperate during the
rhythm and decreased urine D. Urine output of 8 mL in the abuse examination. Place the components of
output past hour assessment in the order the nurse would
C. Prepare a quiet, non- 39. The nurse has received report on 4 perform them.
stimulating and restful pediatric clients on a telemetry unit. Interact with the parent in a friendly
environment Which client should the nurse the nurse is reinforcing teaching to the parent manner, play with the child using a finger
D. Provide soft foods and liberal assess first? of a child diagnosed with ringworm. Which puppet, measure the child's height and
amounts of clear liquids A. Adolescent client with statement by the parent indicates a need for weight, auscultate the child's heart and
36. A 2-month-old recently diagnosed coarctation of the aorta and further teaching? “my child has been lungs, take the child's vital signs
with developmental dysplasia of the diminished femoral pulses infected by a worm and must be treated to
hip (DDH) is beginning treatment B. Infant client with ventricular rid it from the body.” A 3-month-old infant has irritability, facial
with a Pavlik harness. What septal deficit with reported edema, a 1-day history of diarrhea with
instructions should the nurse grunting during feeding The nurse has provided instructions about adequate oral intake, and seizure activity.
provide to the parents? Select all C. Newborn client with patent home care management for the parents of a During assessment, the parents state that they
that apply ductus arteriosus and a loud child diagnosed with rotavirus infection. have recently been diluting formula to save
A. “apply lotion under the straps machine-like systolic murmur Which statements by the parents indicate that money. Which is the most likely cause for the
to protect the skin” D. Preschool client tetralogy of teaching has been effective? Select all that infant’s symptoms?
B. “Dress the child in a shirt Fallot who has finger clubbing apply. Hyponatremia due to water intoxication
and knee socks under the and irritability “hand washing is extremely important in
straps” 40. A nurse auscultates a loud cardiac slowing the spread of rotavirus.” The nurse is teaching the parents of a child
C. “Lightly massage the skin murmur on a newborn with “I will observe my client for decreased diagnosed with cystic fibrosis will advise the
under the straps daily” suspected trisomy 21 (Down urination and dry mucous membranes.” parents to choose foods that satisfy which
D. “Place the diaper under the syndrome). A genetic screen and an “my child can spread the infection with recommended diet?
straps” electrocardiogram are scheduled contaminated hands, toys, and food.” High calorie, high protein, high fat
E. “Remove the harness during that day. The neonates’ vital signs
diaper changes” are shown in the exhibit. What A school nurse is educating the parent of a The school nurse is teaching a class of 10-
37. The nurse is performing well-child should be an appropriate action for young client with pediculosis capitis. Which year-old children about prevention of dental
examinations in a pediatric clinic. the nurse to complete next? statement by the parent indicates caries. Which recommendations would be
Which finding requires further A. Call the healthcare provider understanding of the teaching? “I will part of the nurses teaching plan? Select all
evaluation? (HCP) immediately launder recently won clothing, sheets, and that apply.
A. Bilateral bowlegs (genu B. Document the assessment towels in hot water.” Chew sugar-free gum
varum) in a 15-month-old findings The nurse plans care for a 3-year-old who is Include milk, yogurt, and cheese in dietary
B. Chest rounded with the C. Place the neonate in a knee- admitted with suspected pertussis infection. intake
anteroposterior diameter equal chest position Which instructions will the nurse include in Minimize consumption of sweet, sticky
to the lateral diameter in an D. Provide oxygen to the neonate the plan of care? Select all that apply. foods
infant 41. The nurse is caring for an infant Institute droplet precautions, monitor for Rinse mouth with water after meals when
with osteogenesis imperfecta signs of airway obstruction, offer small brushing is not possible
admitted with a new fracture. The amounts of fluids frequently
1. Reye’s syndrome is a development of clinical manifestation femur to heal in a 3-
rare and severe illness rheumatic fever? of increased ICP in year-old?
affecting children and a. Group A beta- older children? a. 3-8 week
teenagers. Its hemolytic a. Diplopia 10. You have learned that
development has been Streptococcus 8. Which of the following in babies and children
linked with the use of 5. The following are should the nurse with developmental
aspirin and which of considered functions include in the insulin dysplasia (dislocation)
the following? of the Urinary administration of the hip (DDH), the
a. Varicella System, EXCEPT: instruction for the hip joint has not
2. When performing a a. Absorption of parents of a child being formed normally.
physical examination digested discharged on insulin? Which of the following
on an infant, the nurse molecules a. Clean the site is the most common
in charge notes 6. Spina bifida is one of of injection form of DDH?
abnormally low-set the possible neural with soap and a. Subluxation
ears. This finding is tube defects that can water and 11. The long-term
associated with: occur during early avoid alcohol complications seen in
a. Renal embryological 9. Among toddlers and thalassemia major are
anomalies development. Which of children up to age five, associated to which of
3. The nurse is evaluating the following femur fractures usually the following?
a female child with definitions most result from a low a. Hemochromato
acute post accurately describes energy fall. In most sis
streptococcal meningocele? cases, the orthopedic 12. A child was brought to
glomerulonephritis for a. Sac formation surgeon realigns the the emergency
signs of improvement. containing fracture using department with
Which finding typically meninges and fluoroscopy or x-ray complaints of nausea,
is the earliest sign of spinal fluid imaging as a guide and vomiting, fruity-
improvement? 7. Bennett was rushed to immobilizes the leg in a scented breath. The
a. Increased urine the emergency type of cast called a resident on duty
output department with spica cast. diagnosed the child
4. Which of the following possible increased Approximately how with diabetes
organisms is intracranial pressure many weeks does it ketoacidosis. Which of
responsible for the (ICP); which of the take for a fractured the following should
following is an early
the nurse expect to
administer?
a. Normal saline
IV infusion
2. an 8 year old is seen in peds nurse should advance the unequal curvatures in 17. a child with pyloric stenosis is
clinic with symptoms of feeding if which occurs? refractive apparatus having excessive vomiting,
morning stiffness that ANS: D. the infant is taking which of the following is a
improves with activity. The Pedialyte without vomiting or 12. which immunizations should potential complication?
pediatric nurse suspects the desertion the nurse plan to administer ANS: metabolic Alkalosis
child has symptoms of which to normal healthy children
of the following? ANS: B. 7. a child with extensive burns is between the ages of 1 and 5 18. which of the following
juvenile Idiopathic Arthritis scheduled twice a day in a tub years? (select all that apply) instructions would be included
to loosen and remove ANS: DTap, IPV, MMR in the preventive teaching
3. a 4 year old is seem in the ER sloughing tissue. What is the plan about urinary tract
with a fractured of the term used for sloughing burn 13. choose the prevention infections for a preschool
humerus. Which of the would tissue? ANS: Eschar strategies that have female child?
following P’s are assessed to 8. a child is admitted for minimal significantly decreased ANS: A. Wiping front to back
determine the extent of the change nephrotic syndrome. fatalities for children?
child’s injury? The nurse recognizes that the (select all that apply) 20. at which age should the nurse
ANS: C. paralysis and pallor child’s prognosis is related to ANS: Bicycle helmets, car expect an infant to begin
what factor? restraints, smoke detectors smiling in response to
4. a newborn is diagnosed with ANS: D. Response to steroid pleasurable stimuli?
Tracheoesophageal Fistula at therapy. 14. which of the following ANS: 2 months
birth. An initial nursing describes the pathophysiology
function is to assure that 9. a 7 year old child with acute of leukemia? 21. Select the viral communicable
which of the following orders lymphatic leukemia is on ANS: Unrestricted proliferation disease from the following list,
are implemented? steroids. A common side of immature white blood cells which has the classic
ANS: A. Suction as needed effect of corticosteroid (WBCs) symptoms of a centripetal rash
(prednisone) therapy is? seen in 3 stages, fever, anorexia,
5. a 10 year old child is admitted ANS: weight gain 15. a school-age child with celiac and itchiness
A. Rubella
with a hx of crohn disease. disease asks for guidance
10. The school nurse is caring for B. Scarlet fever
The child’s plan of care should about snack that will not
a child with seizures. What is C. Rubeola
include a diet that has which exacerbate the disease, what
the initial clinical D. Varicella
of the following components? snack should the nurse
ANS: B. increased calories manifestation of absence suggest?
seizures that nurse needs to 22. What should a nursing
ANS: D. potato chips intervention to promote parent-
6. after a pyloromyotomy, a be aware of?
infant attachment include?
week old infant is prescribed ANS: Brief loss of consciousness. 16. identify the age group of A. Alleviating stress for parents
feedings of 1 to 2 oz od children per Erickson in the by decreasing their
Pedialyte every 3 hours then 11. a child’s chart indicated developmental stage of participation in the infant’s
advance to breastmilk as Astigmatism. For what should identity vs role confusion ANS: care
tolerated postoperatively. The the nurse assess? ANS: C. 13-21
B. Delaying parent infant myelomeningocele in the B. 50% of a personal best and A. Encourage elevation and
interactions until the second preoperative stage is which? needs immediate emergency application of ice to the
period of reactivity A. Apply a heat lamp to facilitate bronchodilators involved joint
C. Explaining normalcy of drying and toughening of the C. 80% of a personal best, and the B. Administer nonsteroidal anti-
specific infant behaviors to sac routine treatment plan can be inflammatory drugs (NSAIDs)
parents B. Watch for signs that might followed C. Administer DDVAP (synthetic
D. Encouraging parents to hold indicate developing D. 50% to 79% of a personal vasopressin)
the infant frequently unless the hydrocephalus best and needs an increase in the D. Provide intravenous (IV)
infant is fussy C. Place the infant in the prone usual therapy infusion of factor VII
position concentrates
23. A 2-year-old is scheduled to D. Take vital signs every hour 29. An infant in the NICU was
have a tonsillectomy. How would born with hypospadias, which of 32. A parent asks the nurse what
you educate the patient? 26. What statement is true the following should be avoided would be the first indication that
A. Describe the procedure as it is concerning osteogenesis when a child has such condition? acute glomerulonephritis was
about to be performed imperfecta (OI)? A. Catherization improving. What should be the
B. Engage in conversations about A. Later onset disease usually runs B. Intravenous pyelography (IVP) nurse’s best response?
the procedure and encourage a more difficult course C. Circumcision A. Blood pressure will stabilize
the child to express his or her B. It’s a disorder with varying D. Surgery B. Your child will have more
thoughts degrees of bone fragility energy
C. Provide a video and explain in C. It is easily treated 30. A child has had a short-arm C. Urine will be free of protein
clear terms D. Braces and exercises are of no synthetic cast applied. What D. Urine output will increase
D. Use picture books & puppets therapeutic value’ should the nurse teach to the child
and repeat explanations and parents about cast care? 33. Which of the following
27. Evidence-based practice in A. When the cast becomes loose, symptoms are usually seen in the
24. What statement best describes clinical management is grounded healing has begun child with Rubeola (Measles)?
iron deficiency in infants? in which of the following? B. Observe the fingers for any A. Kolpik spots B. Rash
A. It is caused by depression of A. Systematic reviews evidence of discoloration C. appearing on 3rd day C.
the hematopoietic system B. Clinical research studies Elevate the arm when resting Conjunctivitis
B. Diagnosis is easily made C. Expert opinions D. Child may resume swimming D. Slapped cheek appearance
because of the infant’s D. Clinical practice guidelines after the cast dries E. Bull’s neck
emaciated appearance E. Do not allow the child to put
C. It results from a decreased 28. A school-age child has anything inside the cast 36. Which assessments are
intake of milk and the asthma. The nurse should included in the Apgar scoring
premature addition of solid teach the child that if a peak 31. The nurse is teaching the system? (select all that apply)
foods D. Hemoglobin levels expiratory flow rate is the family of a child, age 8 years, with A. Blood glucose
below the normal range yellow zone, this means that moderate hemophilia about home B. Muscle tone
the asthma control is what? care. What should the nurse the C. Blood pressure
25. One of the most important A. Less than 50% of a family to do to minimize joint D. Heart rate E. Respiratory
nursing interventions when caring personal best and needs injury? effort
for an infant with immediate hospitalization
37. A child with severe D. Directly observed therapy disease and its management. Lack of bowel control, flaccid,
anemia requires a unit of red Which of the following states by partial paralysis of lower
blood cells (RBC’s). The 40. A young girl has just injured the mother indicates successful extremites, overflow
nurse explains that the her ankle at school. In addition to teaching? incontinence with constant
transfusion is necessary for notifying the child’s parents, what My son will probably be unable dribbling of urine
which reason. A. Increase the is the most appropriate, immediate to walk independently by the
amount of oxygen available action by the school nurse? time he is 9 to 11 years old. 53. When caring for the child with
to tissues A. Obtain parental permission for Kawasaki disease, what should
B. Allow her parents to come with administration of acetaminophen 47. A child has been admitted to the the nurse know to provide safe
her or aspirin hospital with a blood lead level of and effective care?
C. Fight the infection that she has B. Apply ice 42 mcg/dL. What treatment Therapeutic management
now C. Observe for edema or should the nurse anticipate? includes administration of
D. Help her body stop bleeding by discoloration Initiation of chelation therapy gamma globulin and salicylates.
forming a clot (scab) D. Encourage child to assume a
position of comfort 48. Please select from the 54. What action should the school
38. An 18-month-old child following list the two basic nurse take for a child who has a
is seen in the clinic with 41. A 5-year-old male is concepts used by pediatric nurses hematoma (black eye) with no
Otitis media (OM). Oral diagnosed with diarrhea and mild to foster family centered care with hemorrhage into the anterior
amoxicillin is prescribed dehydration. Which of the children and their families: chamber?
instructions to the parent following oral rehydration enabling and empowerment Apply ice for the first 24 hours.
should include which of the solution (ORS) should be given by
following? A. Stop giving the nurse or parent to replace the 51. A pediatric nurse plans to
medication and come to the stool losses? 55. A health care provider prescribes
educate the parents of a 2- osmitrol (mannitol), 0.5 g/kg
clinic if fever is still present A. 50-100 mL/each stool yeard old child who has intravenously now for a child with
in 24 hours B. 250-275 mL/stool simple constipation. Which minimal change nephrotic
B. Immediately stop giving the C. 125-150 mL/stool of the following info should syndrome. The child weighs 10
medication of hearing loss D. 150-250 mL/stool be included in the teaching? kg. The medication label states,
develops
Include more fibers and “Osmitrol (mannitol) 20g/100
C. Continue medication until all 44. The mother of a 6-month-old fluids ml.” The nurse prepares to
symptoms subside infant has returned to work and is
D. Medication may take 24 to 48 administer the dose. How many
expressing breast milk to be 52. The nurse is preparing to
hours to make symptoms milliliters will the nurse prepare to
frozen. She asks for directions on admit a 5-year old with spina
subside administer the dose?
how to safely thaw the breast milk bifida cystica that was below
in the microwave. What should 25
the second lumbar vertebra.
39. Which of the following the nurse recommend?
What clinical manifestations 56. In a child with sickle cell
therapy is usually ordered for Do not thaw or heat breast milk
of spina bifida cystica below anemia, adequate hydration is
patients with active TB as part of in a microwave oven.
the second lumbar vertebra essential to minimize sickling and
their medication management
should the nurse expect to delay the vasoocclusion and
plan?
observe? SATA. hypoxia-ischemia cycle. What
A. Monotherapy 46. The nurse teaches the mother of a
B. Occupational therapy information should the nurse share
young child with Duchenne’s with parents in a teaching plan?
C. Physical therapy muscular dystrophy about the
Monitor child for signs of 63. A 5-year old is seen in the 67. An 18-month old male is seborrheic dermatitis (cradle cap)
dehydration urgent care clinic with the scheduled for surgery to reposition to shampoo the child’s hair?
7. At what age is it safe to give following history and his undescended tests. What is the Use a soft brush or fine tooth
infants whole milk instead of symptoms; sudden onset of child’s diagnosis? comb to gently scrub the area
commercial infant formula? 12 severe sore throat after going Cryptorchidism (failure for
months to bed, drooling and difficulty testes to descend) 73. A 6-year old child in the ED is
swallowing, axillary diagnosed with Lyme Disease.
8. Exposure to which of the temperature of 102.2F, clear 69. A child has been diagnosed The nurse anticipates that the
following communicable breath sounds, and absence of with a Wilms tumor. What should health care provider orders will
diseases pose a serious threat to cough. The child appears preoperative nursing care include? include the administration of?
fetuses during the first trimester anxious and is flushed. Based Careful bathing and handling amoxicillin
of pregnancy? on these symptoms and
Rubella history, the nurse anticipates a 68. Of the statements below, 74. Verbal informed consent via the
diagnosis of? Acute epiglottis what statement best telephone for a minor child is
9. A pediatrics nurse is taking the represents infectious appropriate when the parent is not
history of a child hospitalized 64. Children with salt losing type mononucleosis? The available in person. Which of the
with Reye syndrome. Which of of Congenital Adrenal casusative pathogen is the following is true in this case? The
the following is a common hyperplasia (CAH) need Epstein barr virus (EBV) verbal consent is required to be
occurrence seen in patients with which of the following in the presence of two witnesses.
Reye syndrome? Fatty changes replacement therapy? 70. A child diagnosed with
in liver aldosterone Coarctation of aorta is 75. The Cochlear implantation is a
scheduled for a follow up safe surgical technique used to
0. A 7-month old infant seen in peds visit. While assessing, the treat which of the following types
65. Of the following, which are
clinic is diagnosed with Isotonic pediatric nurse would expect of hearing loss in children?
possible signs of cerebral
Dehydration. The pediatric nurse to find which of the following sensorineural
palsy? (not sure)
prepares to administer which of symptom? Bounding pulses
Feeding difficulties, poor head
the following fluids that will be in the upper extremities
control after age 3 months,
the most appropriate to rehydrate persistent primitive reflexes
this infant? 71. Select the developmental
0.9% NaCl behaviors usually exhibited
66. A nurse is providing
by an infant (birth- 1 year).
anticipatory guidance to the
61. An adolescent with parents of a toddler. Which of the SATA
diabetes is admitted to the following should the nurse include head turns to locate sounds at 3
ER for treatment of in the teaching? months
hyperglycemia and Develop food habits that will crawls by 10 months
pneumonia. What are the prevent dental caries, expression moro reflex disappears by 4
clinical manifestations of of bedtime fears is common, months
diabetic hyperglycemia? expect negative behaviors plays peek a boo after 4 months
Thirst, fatigue, and associate with negativism and
frequent urination ritualism 72. How should a nurse instruct
the parent of a 3-month old with

You might also like